Programs & Examples On #Missing symbols

Is it possible to access to google translate api for free?

Yes, you can use GT for free. See the post with explanation. And look at repo on GitHub.

UPD 19.03.2019 Here is a version for browser on GitHub.

JavaScript/jQuery to download file via POST with JSON data

In short, there is no simpler way. You need to make another server request to show PDF file. Al though, there are few alternatives but they are not perfect and won't work on all browsers:

  1. Look at data URI scheme. If binary data is small then you can perhaps use javascript to open window passing data in URI.
  2. Windows/IE only solution would be to have .NET control or FileSystemObject to save the data on local file system and open it from there.

How to wait in bash for several subprocesses to finish and return exit code !=0 when any subprocess ends with code !=0?

Trapping CHLD signal may not work because you can lose some signals if they arrived simultaneously.

#!/bin/bash

trap 'rm -f $tmpfile' EXIT

tmpfile=$(mktemp)

doCalculations() {
    echo start job $i...
    sleep $((RANDOM % 5)) 
    echo ...end job $i
    exit $((RANDOM % 10))
}

number_of_jobs=10

for i in $( seq 1 $number_of_jobs )
do
    ( trap "echo job$i : exit value : \$? >> $tmpfile" EXIT; doCalculations ) &
done

wait 

i=0
while read res; do
    echo "$res"
    let i++
done < "$tmpfile"

echo $i jobs done !!!

Difference between document.addEventListener and window.addEventListener?

The document and window are different objects and they have some different events. Using addEventListener() on them listens to events destined for a different object. You should use the one that actually has the event you are interested in.

For example, there is a "resize" event on the window object that is not on the document object.

For example, the "DOMContentLoaded" event is only on the document object.

So basically, you need to know which object receives the event you are interested in and use .addEventListener() on that particular object.

Here's an interesting chart that shows which types of objects create which types of events: https://developer.mozilla.org/en-US/docs/DOM/DOM_event_reference


If you are listening to a propagated event (such as the click event), then you can listen for that event on either the document object or the window object. The only main difference for propagated events is in timing. The event will hit the document object before the window object since it occurs first in the hierarchy, but that difference is usually immaterial so you can pick either. I find it generally better to pick the closest object to the source of the event that meets your needs when handling propagated events. That would suggest that you pick document over window when either will work. But, I'd often move even closer to the source and use document.body or even some closer common parent in the document (if possible).

How to iterate object keys using *ngFor

1.Create a custom pipe to get keys.

import { Pipe, PipeTransform } from '@angular/core';

@Pipe({
  name: 'keys'
})
export class KeysPipe implements PipeTransform {

  transform(value: any, args?: any): any {
    return Object.keys(value);
  }
}
  1. In angular template file, you can use *ngFor and iterate over your object object
<div class ="test" *ngFor="let key of Obj | keys">
    {{key}}
    {{Obj[key].property}
<div>

Oracle "Partition By" Keyword

I think, this example suggests a small nuance on how the partitioning works and how group by works. My example is from Oracle 12, if my example happens to be a compiling bug.

I tried :

SELECT t.data_key
,      SUM ( CASE when t.state = 'A' THEN 1 ELSE 0 END) 
OVER   (PARTITION BY t.data_key) count_a_rows
,      SUM ( CASE when t.state = 'B' THEN 1 ELSE 0 END) 
OVER   (PARTITION BY t.data_key) count_b_rows
,      SUM ( CASE when t.state = 'C' THEN 1 ELSE 0 END) 
OVER   (PARTITION BY t.data_key) count_c_rows
,      COUNT (1) total_rows
from mytable t
group by t.data_key  ---- This does not compile as the compiler feels that t.state isn't in the group by and doesn't recognize the aggregation I'm looking for

This however works as expected :

SELECT distinct t.data_key
,      SUM ( CASE when t.state = 'A' THEN 1 ELSE 0 END) 
OVER   (PARTITION BY t.data_key) count_a_rows
,      SUM ( CASE when t.state = 'B' THEN 1 ELSE 0 END) 
OVER   (PARTITION BY t.data_key) count_b_rows
,      SUM ( CASE when t.state = 'C' THEN 1 ELSE 0 END) 
OVER   (PARTITION BY t.data_key) count_c_rows
,      COUNT (1) total_rows
from mytable t;

Producing the number of elements in each state based on the external key "data_key". So, if, data_key = 'APPLE' had 3 rows with state 'A', 2 rows with state 'B', a row with state 'C', the corresponding row for 'APPLE' would be 'APPLE', 3, 2, 1, 6.

SCRIPT5: Access is denied in IE9 on xmlhttprequest

  $.ajax({
        url: '//freegeoip.net/json/',
        type: 'POST',
        dataType: 'jsonp',
        success: function(location) {
            alert(location.ip);
        }
    });

This code will work https sites too

AngularJS ui router passing data between states without URL

We can use params, new feature of the UI-Router:

API Reference / ui.router.state / $stateProvider

params A map which optionally configures parameters declared in the url, or defines additional non-url parameters. For each parameter being configured, add a configuration object keyed to the name of the parameter.

See the part: "...or defines additional non-url parameters..."

So the state def would be:

$stateProvider
  .state('home', {
    url: "/home",
    templateUrl: 'tpl.html',
    params: { hiddenOne: null, }
  })

Few examples form the doc mentioned above:

// define a parameter's default value
params: {
  param1: { value: "defaultValue" }
}
// shorthand default values
params: {
  param1: "defaultValue",
  param2: "param2Default"
}

// param will be array []
params: {
  param1: { array: true }
}

// handling the default value in url:
params: {
  param1: {
    value: "defaultId",
    squash: true
} }
// squash "defaultValue" to "~"
params: {
  param1: {
    value: "defaultValue",
    squash: "~"
  } }

EXTEND - working example: http://plnkr.co/edit/inFhDmP42AQyeUBmyIVl?p=info

Here is an example of a state definition:

 $stateProvider
  .state('home', {
      url: "/home",
      params : { veryLongParamHome: null, },
      ...
  })
  .state('parent', {
      url: "/parent",
      params : { veryLongParamParent: null, },
      ...
  })
  .state('parent.child', { 
      url: "/child",
      params : { veryLongParamChild: null, },
      ...
  })

This could be a call using ui-sref:

<a ui-sref="home({veryLongParamHome:'Home--f8d218ae-d998-4aa4-94ee-f27144a21238'
  })">home</a>

<a ui-sref="parent({ 
    veryLongParamParent:'Parent--2852f22c-dc85-41af-9064-d365bc4fc822'
  })">parent</a>

<a ui-sref="parent.child({
    veryLongParamParent:'Parent--0b2a585f-fcef-4462-b656-544e4575fca5',  
    veryLongParamChild:'Child--f8d218ae-d998-4aa4-94ee-f27144a61238'
  })">parent.child</a>

Check the example here

Duplicate / Copy records in the same MySQL table

Alex's answer needs some care (e.g. locking or a transaction) in multi-client environments.

Assuming the AUTO ID field is the first one in the table (a usual case), we can make use of implicit transactions.

    CREATE TEMPORARY TABLE tmp SELECT * from invoices WHERE ...;
    ALTER TABLE tmp drop ID; # drop autoincrement field
    # UPDATE tmp SET ...; # just needed to change other unique keys
    INSERT INTO invoices SELECT 0,tmp.* FROM tmp;
    DROP TABLE tmp;

From the MySQL docs:

Using AUTO_INCREMENT: You can also explicitly assign NULL or 0 to the column to generate sequence numbers.

The relationship could not be changed because one or more of the foreign-key properties is non-nullable

I've no idea why the other two answers are so popular!

I believe you were right in assuming the ORM framework should handle it - after all, that is what it promises to deliver. Otherwise your domain model gets corrupted by persistence concerns. NHibernate manages this happily if you setup the cascade settings correctly. In Entity Framework it is also possible, they just expect you to follow better standards when setting up your database model, especially when they have to infer what cascading should be done:

You have to define the parent - child relationship correctly by using an "identifying relationship".

If you do this, Entity Framework knows the child object is identified by the parent, and therefore it must be a "cascade-delete-orphans" situation.

Other than the above, you might need to (from NHibernate experience)

thisParent.ChildItems.Clear();
thisParent.ChildItems.AddRange(modifiedParent.ChildItems);

instead of replacing the list entirely.

UPDATE

@Slauma's comment reminded me that detached entities are another part of the overall problem. To solve that, you can take the approach of using a custom model binder that constructs your models by attempting to load it from the context. This blog post shows an example of what I mean.

Check if DataRow exists by column name in c#?

if (row.Columns.Contains("US_OTHERFRIEND"))

How can I make a jQuery UI 'draggable()' div draggable for touchscreen?

jQuery ui 1.9 is going to take care of this for you. Heres a demo of the pre:

https://dl.dropbox.com/u/3872624/lab/touch/index.html

Just grab the jquery.mouse.ui.js out, stick it under the jQuery ui file you're loading, and that's all you should have to do! Works for sortable as well.

This code is working great for me, but if your getting errors, an updated version of jquery.mouse.ui.js can be found here:

Jquery-ui sortable doesn't work on touch devices based on Android or IOS

How can I open two pages from a single click without using JavaScript?

If you have the authority to edit the pages to be opened, you can href to 'A' page and in the A page you can put link to B page in onpageload attribute of body tag.

Purpose of installing Twitter Bootstrap through npm?

  1. Use npm/bower to install bootstrap if you want to recompile it/change less files/test. With grunt it would be easier to do this, as shown on http://getbootstrap.com/getting-started/#grunt. If you only want to add precompiled libraries feel free to manually include files to project.

  2. No, you have to do this by yourself or use separate grunt tool. For example 'grunt-contrib-concat' How to concatenate and minify multiple CSS and JavaScript files with Grunt.js (0.3.x)

Should CSS always preceed Javascript?

Here is a SUMMARY of all the major answers above (or maybe below later :)

For modern browsers, put css wherever you like it. They would analyze your html file (which they call speculative parsing) and start downloading css in parallel with html parsing.

For old browsers keep putting css on top (if you don't want to show a naked but interactive page first).

For all browsers, put javascript as farther down on the page as possible, since it will halt parsing of your html. Preferably, download it asynchronously (i.e., ajax call)

There are also, some experimental results for a particular case which claims putting javascript first (as opposed to traditional wisdom of putting CSS first) gives better performance but there is no logical reasoning given for it, and lacks validation regarding widespread applicability, so you can ignore it for now.

So, to answer the question: Yes. The recommendation to include the CSS before JS is invalid for the modern browsers. Put CSS wherever you like, and put JS towards the end, as possible.

How do I print uint32_t and uint16_t variables value?

You need to include inttypes.h if you want all those nifty new format specifiers for the intN_t types and their brethren, and that is the correct (ie, portable) way to do it, provided your compiler complies with C99. You shouldn't use the standard ones like %d or %u in case the sizes are different to what you think.

It includes stdint.h and extends it with quite a few other things, such as the macros that can be used for the printf/scanf family of calls. This is covered in section 7.8 of the ISO C99 standard.

For example, the following program:

#include <stdio.h>
#include <inttypes.h>
int main (void) {
    uint32_t a=1234;
    uint16_t b=5678;
    printf("%" PRIu32 "\n",a);
    printf("%" PRIu16 "\n",b);
    return 0;
}

outputs:

1234
5678

Getting today's date in YYYY-MM-DD in Python?

Datetime is just lovely if you like remembering funny codes. Wouldn't you prefer simplicity?

>>> import arrow
>>> arrow.now().format('YYYY-MM-DD')
'2017-02-17'

This module is clever enough to understand what you mean.

Just do pip install arrow.

Addendum: In answer to those who become exercised over this answer let me just say that arrow represents one of the alternative approaches to dealing with dates in Python. That's mostly what I meant to suggest.

How to keep onItemSelected from firing off on a newly instantiated Spinner?

spinner.setSelection(Adapter.NO_SELECTION, false);

Input jQuery get old value before onchange and get value after on change

My business aim was removing classes form previous input and add it to a new one.
In this case there was simple solution: remove classes from all inputs before add

<div>
   <input type="radio" checked><b class="darkred">Value1</b>
   <input type="radio"><b>Value2</b>
   <input type="radio"><b>Value3</b>
</div>

and

$('input[type="radio"]').on('change', function () {
   var current = $(this);
   current.closest('div').find('input').each(function () {
       (this).next().removeClass('darkred')
   });
   current.next().addClass('darkred');
});

JsFiddle: http://jsfiddle.net/gkislin13/tybp8skL

Search and replace part of string in database

You can do it with an UPDATE statement setting the value with a REPLACE

UPDATE
    Table
SET
    Column = Replace(Column, 'find value', 'replacement value')
WHERE
    xxx

You will want to be extremely careful when doing this! I highly recommend doing a backup first.

Why am I getting "Unable to find manifest signing certificate in the certificate store" in my Excel Addin?

Adding new certificate resolved this issue for me. Properties page -> signing -> Click on Create test certificate

LINQ to SQL Left Outer Join

I found 1 solution. if want to translate this kind of SQL (left join) into Linq Entity...

SQL:

SELECT * FROM [JOBBOOKING] AS [t0]
LEFT OUTER JOIN [REFTABLE] AS [t1] ON ([t0].[trxtype] = [t1].[code])
                                  AND ([t1]. [reftype] = "TRX")

LINQ:

from job in JOBBOOKINGs
join r in (from r1 in REFTABLEs where r1.Reftype=="TRX" select r1) 
          on job.Trxtype equals r.Code into join1
from j in join1.DefaultIfEmpty()
select new
{
   //cols...
}

How to compile C program on command line using MinGW?

I had the same problem with .c files that contained functions (not main() of my program). For example, my header files were "fact.h" and "fact.c", and my main program was "main.c" so my commands were like this:

E:\proj> gcc -c fact.c

Now I had an object file of fact.c (fact.o). after that:

E:\proj>gcc -o prog.exe fact.o main.c

Then my program (prog.exe) was ready to use and worked properly. I think that -c after gcc was important, because it makes object files that can attach to make the program we need. Without using -c, gcc ties to find main in your program and when it doesn't find it, it gives you this error.

URL encode sees “&” (ampersand) as “&amp;” HTML entity

There is HTML and URI encodings. &amp; is & encoded in HTML while %26 is & in URI encoding.

So before URI encoding your string you might want to HTML decode and then URI encode it :)

var div = document.createElement('div');
div.innerHTML = '&amp;AndOtherHTMLEncodedStuff';
var htmlDecoded = div.firstChild.nodeValue;
var urlEncoded = encodeURIComponent(htmlDecoded);

result %26AndOtherHTMLEncodedStuff

Hope this saves you some time

What is the difference between server side cookie and client side cookie?

All cookies are client and server

There is no difference. A regular cookie can be set server side or client side. The 'classic' cookie will be sent back with each request. A cookie that is set by the server, will be sent to the client in a response. The server only sends the cookie when it is explicitly set or changed, while the client sends the cookie on each request.

But essentially it's the same cookie.

But, behavior can change

A cookie is basically a name=value pair, but after the value can be a bunch of semi-colon separated attributes that affect the behavior of the cookie if it is so implemented by the client (or server). Those attributes can be about lifetime, context and various security settings.

HTTP-only (is not server-only)

One of those attributes can be set by a server to indicate that it's an HTTP-only cookie. This means that the cookie is still sent back and forth, but it won't be available in JavaScript. Do note, though, that the cookie is still there! It's only a built in protection in the browser, but if somebody would use a ridiculously old browser like IE5, or some custom client, they can actually read the cookie!

So it seems like there are 'server cookies', but there are actually not. Those cookies are still sent to the client. On the client there is no way to prevent a cookie from being sent to the server.

Alternatives to achieve 'only-ness'

If you want to store a value only on the server, or only on the client, then you'd need some other kind of storage, like a file or database on the server, or Local Storage on the client.

Populate a datagridview with sql query results

Years late but here's the simplest for others in case.

String connectionString = @"Data Source=LOCALHOST;Initial Catalog=DB;Integrated Security=true";

SqlConnection cnn = new SqlConnection(connectionString);
SqlDataAdapter sda = new SqlDataAdapter("SELECT * FROM tblEmployee;", cnn);

DataTable data = new DataTable();
sda.Fill(data);

DataGridView1.DataSource = data;

Using DataSet is not necessary and DataTable should be good enough. SQLCommandBuilder is unnecessary either.

What is the Java equivalent of PHP var_dump?

It is not quite as baked-in in Java, so you don't get this for free. It is done with convention rather than language constructs. In all data transfer classes (and maybe even in all classes you write...), you should implement a sensible toString method. So here you need to override toString() in your Person class and return the desired state.

There are utilities available that help with writing a good toString method, or most IDEs have an automatic toString() writing shortcut.

What is the difference between the float and integer data type when the size is the same?

  • float stores floating-point values, that is, values that have potential decimal places
  • int only stores integral values, that is, whole numbers

So while both are 32 bits wide, their use (and representation) is quite different. You cannot store 3.141 in an integer, but you can in a float.

Dissecting them both a little further:

In an integer, all bits are used to store the number value. This is (in Java and many computers too) done in the so-called two's complement. This basically means that you can represent the values of −231 to 231 − 1.

In a float, those 32 bits are divided between three distinct parts: The sign bit, the exponent and the mantissa. They are laid out as follows:

S EEEEEEEE MMMMMMMMMMMMMMMMMMMMMMM

There is a single bit that determines whether the number is negative or non-negative (zero is neither positive nor negative, but has the sign bit set to zero). Then there are eight bits of an exponent and 23 bits of mantissa. To get a useful number from that, (roughly) the following calculation is performed:

M × 2E

(There is more to it, but this should suffice for the purpose of this discussion)

The mantissa is in essence not much more than a 24-bit integer number. This gets multiplied by 2 to the power of the exponent part, which, roughly, is a number between −128 and 127.

Therefore you can accurately represent all numbers that would fit in a 24-bit integer but the numeric range is also much greater as larger exponents allow for larger values. For example, the maximum value for a float is around 3.4 × 1038 whereas int only allows values up to 2.1 × 109.

But that also means, since 32 bits only have 4.2 × 109 different states (which are all used to represent the values int can store), that at the larger end of float's numeric range the numbers are spaced wider apart (since there cannot be more unique float numbers than there are unique int numbers). You cannot represent some numbers exactly, then. For example, the number 2 × 1012 has a representation in float of 1,999,999,991,808. That might be close to 2,000,000,000,000 but it's not exact. Likewise, adding 1 to that number does not change it because 1 is too small to make a difference in the larger scales float is using there.

Similarly, you can also represent very small numbers (between 0 and 1) in a float but regardless of whether the numbers are very large or very small, float only has a precision of around 6 or 7 decimal digits. If you have large numbers those digits are at the start of the number (e.g. 4.51534 × 1035, which is nothing more than 451534 follows by 30 zeroes – and float cannot tell anything useful about whether those 30 digits are actually zeroes or something else), for very small numbers (e.g. 3.14159 × 10−27) they are at the far end of the number, way beyond the starting digits of 0.0000...

Javascript split regex question

You need the put the characters you wish to split on in a character class, which tells the regular expression engine "any of these characters is a match". For your purposes, this would look like:

date.split(/[.,\/ -]/)

Although dashes have special meaning in character classes as a range specifier (ie [a-z] means the same as [abcdefghijklmnopqrstuvwxyz]), if you put it as the last thing in the class it is taken to mean a literal dash and does not need to be escaped.

To explain why your pattern didn't work, /-./ tells the regular expression engine to match a literal dash character followed by any character (dots are wildcard characters in regular expressions). With "02-25-2010", it would split each time "-2" is encountered, because the dash matches and the dot matches "2".

Android java.lang.NoClassDefFoundError

Edit the build path in this order, this worked for me.

Make sure the /gen is before /src

enter image description here

Find a commit on GitHub given the commit hash

View single commit:
https://github.com/<user>/<project>/commit/<hash>

View log:
https://github.com/<user>/<project>/commits/<hash>

View full repo:
https://github.com/<user>/<project>/tree/<hash>

<hash> can be any length as long as it is unique.

Slicing of a NumPy 2d array, or how do I extract an mxm submatrix from an nxn array (n>m)?

I don't think that x[[1,3]][:,[1,3]] is hardly readable. If you want to be more clear on your intent, you can do:

a[[1,3],:][:,[1,3]]

I am not an expert in slicing but typically, if you try to slice into an array and the values are continuous, you get back a view where the stride value is changed.

e.g. In your inputs 33 and 34, although you get a 2x2 array, the stride is 4. Thus, when you index the next row, the pointer moves to the correct position in memory.

Clearly, this mechanism doesn't carry well into the case of an array of indices. Hence, numpy will have to make the copy. After all, many other matrix math function relies on size, stride and continuous memory allocation.

How to add 10 minutes to my (String) time?

Something like this

 String myTime = "14:10";
 SimpleDateFormat df = new SimpleDateFormat("HH:mm");
 Date d = df.parse(myTime); 
 Calendar cal = Calendar.getInstance();
 cal.setTime(d);
 cal.add(Calendar.MINUTE, 10);
 String newTime = df.format(cal.getTime());

As a fair warning there might be some problems if daylight savings time is involved in this 10 minute period.

How do you get an iPhone's device name

To get an iPhone's device name programmatically

 UIDevice *deviceInfo = [UIDevice currentDevice];

 NSLog(@"Device name:  %@", deviceInfo.name); 

// Device name: my iPod

Understanding the results of Execute Explain Plan in Oracle SQL Developer

In recent Oracle versions the COST represent the amount of time that the optimiser expects the query to take, expressed in units of the amount of time required for a single block read.

So if a single block read takes 2ms and the cost is expressed as "250", the query could be expected to take 500ms to complete.

The optimiser calculates the cost based on the estimated number of single block and multiblock reads, and the CPU consumption of the plan. the latter can be very useful in minimising the cost by performing certain operations before others to try and avoid high CPU cost operations.

This raises the question of how the optimiser knows how long operations take. recent Oracle versions allow the collections of "system statistics", which are definitely not to be confused with statistics on tables or indexes. The system statistics are measurements of the performance of the hardware, mostly importantly:

  1. How long a single block read takes
  2. How long a multiblock read takes
  3. How large a multiblock read is (often different to the maximum possible due to table extents being smaller than the maximum, and other reasons).
  4. CPU performance

These numbers can vary greatly according to the operating environment of the system, and different sets of statistics can be stored for "daytime OLTP" operations and "nighttime batch reporting" operations, and for "end of month reporting" if you wish.

Given these sets of statistics, a given query execution plan can be evaluated for cost in different operating environments, which might promote use of full table scans at some times or index scans at others.

The cost is not perfect, but the optimiser gets better at self-monitoring with every release, and can feedback the actual cost in comparison to the estimated cost in order to make better decisions for the future. this also makes it rather more difficult to predict.

Note that the cost is not necessarily wall clock time, as parallel query operations consume a total amount of time across multiple threads.

In older versions of Oracle the cost of CPU operations was ignored, and the relative costs of single and multiblock reads were effectively fixed according to init parameters.

How to alter a column's data type in a PostgreSQL table?

If data already exists in the column you should do:

ALTER TABLE tbl_name ALTER COLUMN col_name TYPE integer USING col_name::integer;

As pointed out by @nobu and @jonathan-porter in comments to @derek-kromm's answer.

How to make Apache serve index.php instead of index.html?

As others have noted, most likely you don't have .html set up to handle php code.

Having said that, if all you're doing is using index.html to include index.php, your question should probably be 'how do I use index.php as index document?

In which case, for Apache (httpd.conf), search for DirectoryIndex and replace the line with this (will only work if you have dir_module enabled, but that's default on most installs):

DirectoryIndex index.php

If you use other directory indexes, list them in order of preference i.e.

DirectoryIndex index.php index.phtml index.html index.htm

C# how to create a Guid value?

If you want to create a "desired" Guid you can do

var tempGuid = Guid.Parse("<guidValue>");

where <guidValue> would be something like 1A3B944E-3632-467B-A53A-206305310BAE.

Convert all strings in a list to int

A little bit more expanded than list comprehension but likewise useful:

def str_list_to_int_list(str_list):
    n = 0
    while n < len(str_list):
        str_list[n] = int(str_list[n])
        n += 1
    return(str_list)

e.g.

>>> results = ["1", "2", "3"]
>>> str_list_to_int_list(results)
[1, 2, 3]

Also:

def str_list_to_int_list(str_list):
    int_list = [int(n) for n in str_list]
    return int_list

Is it valid to define functions in JSON results?

JSON explicitly excludes functions because it isn't meant to be a JavaScript-only data structure (despite the JS in the name).

How to go from one page to another page using javascript?

You cannot sanely depend on client side JavaScript to determine if user credentials are correct. The browser (and all code that executes that) is under the control of the user, not you, so it is not trustworthy.

The username and password need to be entered using a form. The OK button will be a submit button. The action attribute must point to a URL which will be handled by a program that checks the credentials.

This program could be written in JavaScript, but how you go about that would depend on which server side JavaScript engine you were using. Note that SSJS is not a mainstream technology so if you really want to use it, you would have to use specialised hosting or admin your own server.

(Half a decade later and SSJS is much more common thanks to Node.js, it is still fairly specialised though).

If you want to redirect afterwards, then the program needs to emit an HTTP Location header.

Note that you need to check the credentials are OK (usually by storing a token, which isn't the actual password, in a cookie) before outputting any private page. Otherwise anyone could get to the private pages by knowing the URL (and thus bypassing the login system).

Proper use of 'yield return'

I tend to use yield-return when I calculate the next item in the list (or even the next group of items).

Using your Version 2, you must have the complete list before returning. By using yield-return, you really only need to have the next item before returning.

Among other things, this helps spread the computational cost of complex calculations over a larger time-frame. For example, if the list is hooked up to a GUI and the user never goes to the last page, you never calculate the final items in the list.

Another case where yield-return is preferable is if the IEnumerable represents an infinite set. Consider the list of Prime Numbers, or an infinite list of random numbers. You can never return the full IEnumerable at once, so you use yield-return to return the list incrementally.

In your particular example, you have the full list of products, so I'd use Version 2.

How to prune local tracking branches that do not exist on remote anymore

check for targets

for target in $(git branch | grep -Eiv "master|develop|branchYouWantToLive"); do echo $target; done

run with for & subcommands

for target in $(git branch | grep -Eiv "master|develop|branchYouWantToLive"); do git branch -D $target; done

you can extend other something works about branches.

Select N random elements from a List<T> in C#

Using LINQ with large lists (when costly to touch each element) AND if you can live with the possibility of duplicates:

new int[5].Select(o => (int)(rnd.NextDouble() * maxIndex)).Select(i => YourIEnum.ElementAt(i))

For my use i had a list of 100.000 elements, and because of them being pulled from a DB I about halfed (or better) the time compared to a rnd on the whole list.

Having a large list will reduce the odds greatly for duplicates.

SQL MERGE statement to update data

UPDATE ed
SET ed.kWh = ted.kWh
FROM energydata ed
INNER JOIN temp_energydata ted ON ted.webmeterID = ed.webmeterID

What are the RGB codes for the Conditional Formatting 'Styles' in Excel?

Copy conditionally formatted cells into Word (using CTRL+C, CTRL+V). Copy them back into Excel, keeping the source formatting. Now the conditional formatting is lost but you still have the colors and can check the RGB choosing Home > Fill color (or Font color) > More colors.

C++, copy set to vector

You need to use a back_inserter:

std::copy(input.begin(), input.end(), std::back_inserter(output));

std::copy doesn't add elements to the container into which you are inserting: it can't; it only has an iterator into the container. Because of this, if you pass an output iterator directly to std::copy, you must make sure it points to a range that is at least large enough to hold the input range.

std::back_inserter creates an output iterator that calls push_back on a container for each element, so each element is inserted into the container. Alternatively, you could have created a sufficient number of elements in the std::vector to hold the range being copied:

std::vector<double> output(input.size());
std::copy(input.begin(), input.end(), output.begin());

Or, you could use the std::vector range constructor:

std::vector<double> output(input.begin(), input.end()); 

Find object by its property in array of objects with AngularJS way

The solucion that work for me is the following

$filter('filter')(data, {'id':10}) 

How does one add keyboard languages and switch between them in Linux Mint 16?

For Linux (I am using Fedora 30) the Shortcut is (Window/Start + Space) Try that and tell me. That works for me

version `CXXABI_1.3.8' not found (required by ...)

GCC 4.9 introduces a newer C++ ABI version than your system libstdc++ has, so you need to tell the loader to use this newer version of the library by adding that path to LD_LIBRARY_PATH. Unfortunately, I cannot tell you straight off where the libstdc++ so for your GCC 4.9 installation is located, as this depends on how you configured GCC. So you need something in the style of:

export LD_LIBRARY_PATH=/home/user/lib/gcc-4.9.0/lib:/home/user/lib/boost_1_55_0/stage/lib:$LD_LIBRARY_PATH

Note the actual path may be different (there might be some subdirectory hidden under there, like `x86_64-unknown-linux-gnu/4.9.0´ or similar).

Only allow specific characters in textbox

Intercept the KeyPressed event is in my opinion a good solid solution. Pay attention to trigger code characters (e.KeyChar lower then 32) if you use a RegExp.

But in this way is still possible to inject characters out of range whenever the user paste text from the clipboard. Unfortunately I did not found correct clipboard events to fix this.

So a waterproof solution is to intercept TextBox.TextChanged. Here is sometimes the original out of range character visible, for a short time. I recommend to implement both.

using System.Text.RegularExpressions;

private void Form1_Shown(object sender, EventArgs e)
{
    filterTextBoxContent(textBox1);
}


string pattern = @"[^0-9^+^\-^/^*^(^)]";

private void textBox1_KeyPress(object sender, KeyPressEventArgs e)
{
    if(e.KeyChar >= 32 && Regex.Match(e.KeyChar.ToString(), pattern).Success) { e.Handled = true; }
}

private void textBox1_TextChanged(object sender, EventArgs e)
{
    filterTextBoxContent(textBox1);
}

private bool filterTextBoxContent(TextBox textBox)
{
    string text = textBox.Text;

    MatchCollection matches = Regex.Matches(text, pattern);
    bool matched = false;

    int selectionStart = textBox.SelectionStart;
    int selectionLength = textBox.SelectionLength;

    int leftShift = 0;
    foreach (Match match in matches)
    {
        if (match.Success && match.Captures.Count > 0)
        {
            matched = true;
            Capture capture = match.Captures[0];

            int captureLength = capture.Length;
            int captureStart = capture.Index - leftShift;
            int captureEnd = captureStart + captureLength;

            int selectionEnd = selectionStart + selectionLength;

            text = text.Substring(0, captureStart) + text.Substring(captureEnd, text.Length - captureEnd);

            textBox.Text = text;

            int boundSelectionStart = selectionStart < captureStart ? -1 : (selectionStart < captureEnd ? 0 : 1);
            int boundSelectionEnd = selectionEnd < captureStart ? -1 : (selectionEnd < captureEnd ? 0 : 1);

            if (boundSelectionStart == -1)
            {
                if (boundSelectionEnd == 0)
                {
                    selectionLength -= selectionEnd - captureStart;
                }
                else if (boundSelectionEnd == 1)
                {
                    selectionLength -= captureLength;
                }
            }
            else if (boundSelectionStart == 0)
            {
                if (boundSelectionEnd == 0)
                {
                    selectionStart = captureStart;
                    selectionLength = 0;
                }
                else if (boundSelectionEnd == 1)
                {
                    selectionStart = captureStart;
                    selectionLength -= captureEnd - selectionStart;
                }
            }
            else if (boundSelectionStart == 1)
            {
                selectionStart -= captureLength;
            }

            leftShift++;
        }
    }

    textBox.SelectionStart = selectionStart;
    textBox.SelectionLength = selectionLength;

    return matched;
}

How to collapse blocks of code in Eclipse?

Preferences -> C++ -> Editor -> Folding ?

Make a right click in the editor window and go to preferences there, then only the editor-relevant section of the preferences dialog will appear. This works for JDT, CDT etc...

Converting char* to float or double

You are missing an include : #include <stdlib.h>, so GCC creates an implicit declaration of atof and atod, leading to garbage values.

And the format specifier for double is %f, not %d (that is for integers).

#include <stdlib.h>
#include <stdio.h>

int main()
{
  char *test = "12.11";
  double temp = strtod(test,NULL);
  float ftemp = atof(test);
  printf("price: %f, %f",temp,ftemp);
  return 0;
}
/* Output */
price: 12.110000, 12.110000

chai test array equality doesn't work as expected

For expect, .equal will compare objects rather than their data, and in your case it is two different arrays.

Use .eql in order to deeply compare values. Check out this link.
Or you could use .deep.equal in order to simulate same as .eql.
Or in your case you might want to check .members.

For asserts you can use .deepEqual, link.

Seaborn Barplot - Displaying Values

Works with single ax or with matrix of ax (subplots)

from matplotlib import pyplot as plt
import numpy as np

def show_values_on_bars(axs):
    def _show_on_single_plot(ax):        
        for p in ax.patches:
            _x = p.get_x() + p.get_width() / 2
            _y = p.get_y() + p.get_height()
            value = '{:.2f}'.format(p.get_height())
            ax.text(_x, _y, value, ha="center") 

    if isinstance(axs, np.ndarray):
        for idx, ax in np.ndenumerate(axs):
            _show_on_single_plot(ax)
    else:
        _show_on_single_plot(axs)

fig, ax = plt.subplots(1, 2)
show_values_on_bars(ax)

Show compose SMS view in Android

This worked for me.

public void onCreate(Bundle savedInstanceState) {
    super.onCreate(savedInstanceState);
    setContentView(R.layout.main);
    Button btnSendSMS = (Button) findViewById(R.id.btnSendSMS);
    btnSendSMS.setOnClickListener(new View.OnClickListener() {
        public void onClick(View v) {
            sendSMS("5556", "Hi You got a message!");
           /*here i can send message to emulator 5556. In Real device 
            *you can change number*/
        }
    });
}

//Sends an SMS message to another device

private void sendSMS(String phoneNumber, String message) {
    SmsManager sms = SmsManager.getDefault();
    sms.sendTextMessage(phoneNumber, null, message, null, null);
}

You can add this line in AndroidManifest.xml

<uses-permission android:name="android.permission.SEND_SMS"/>

Take a look at this

This may be helpful for you.

Set Locale programmatically

@SuppressWarnings("deprecation")
public static void forceLocale(Context context, String localeCode) {
    String localeCodeLowerCase = localeCode.toLowerCase();

    Resources resources = context.getApplicationContext().getResources();
    Configuration overrideConfiguration = resources.getConfiguration();
    Locale overrideLocale = new Locale(localeCodeLowerCase);

    if (Build.VERSION.SDK_INT >= Build.VERSION_CODES.JELLY_BEAN_MR1) {
        overrideConfiguration.setLocale(overrideLocale);
    } else {
        overrideConfiguration.locale = overrideLocale;
    }

    if (Build.VERSION.SDK_INT >= Build.VERSION_CODES.N) {
        context.getApplicationContext().createConfigurationContext(overrideConfiguration);
    } else {
        resources.updateConfiguration(overrideConfiguration, null);
    }
}

Just use this helper method to force specific locale.

UDPATE 22 AUG 2017. Better use this approach.

SUM OVER PARTITION BY

In my opinion, I think it's important to explain the why behind the need for a GROUP BY in your SQL when summing with OVER() clause and why you are getting repeated lines of data when you are expecting one row per BrandID.

Take this example: You need to aggregate the total sale price of each order line, per specific order category, between two dates, but you also need to retain individual order data in your final results. A SUM() on the SalesPrice column would not allow you to get the correct totals because it would require a GROUP BY, therefore squashing the details because you wouldn't be able to keep the individual order lines in the select statement.

Many times we see a #temp table, @table variable, or CTE filled with the sum of our data and grouped up so we can join to it again later to get a column of the sums we need. This can add processing time and extra lines of code. Instead, use OVER(PARTITION BY ()) like this:

SELECT
  OrderLine, 
  OrderDateTime, 
  SalePrice, 
  OrderCategory,
  SUM(SalePrice) OVER(PARTITION BY OrderCategory) AS SaleTotalPerCategory
FROM tblSales 
WHERE OrderDateTime BETWEEN @StartDate AND @EndDate

Notice we are not grouping and we have individual order lines column selected. The PARTITION BY in the last column will return us a sales price total for each row of data in each category. What the last column essentially says is, we want the sum of the sale price (SUM(SalePrice)) over a partition of my results and by a specified category (OVER(PARTITION BY CategoryHere)).

If we remove the other columns from our select statement, and leave our final SUM() column, like this:

SELECT
  SUM(SalePrice) OVER(PARTITION BY OrderCategory) AS SaleTotalPerCategory
FROM tblSales 
WHERE OrderDateTime BETWEEN @StartDate AND @EndDate

The results will still repeat this sum for each row in our original result set. The reason is this method does not require a GROUP BY. If you don't need to retain individual line data, then simply SUM() without the use of OVER() and group up your data appropriately. Again, if you need an additional column with specific totals, you can use the OVER(PARTITION BY ()) method described above without additional selects to join back to.

The above is purely for explaining WHY he is getting repeated lines of the same number and to help understand what this clause provides. This method can be used in many ways and I highly encourage further reading from the documentation here:

Over Clause

Mail not sending with PHPMailer over SSL using SMTP

Don't use SSL on port 465, it's been deprecated since 1998 and is only used by Microsoft products that didn't get the memo; use TLS on port 587 instead: So, the code below should work very well for you.

mail->IsSMTP(); // telling the class to use SMTP
$mail->Host       = "smtp.gmail.com"; // SMTP server

$mail->SMTPAuth   = true;                  // enable SMTP authentication
$mail->SMTPSecure = "tls";                 // sets the prefix to the servier
$mail->Host       = "smtp.gmail.com";      // sets GMAIL as the SMTP server
$mail->Port       = 587;                   // set the SMTP port for the 

How to write super-fast file-streaming code in C#?

No one suggests threading? Writing the smaller files looks like text book example of where threads are useful. Set up a bunch of threads to create the smaller files. this way, you can create them all in parallel and you don't need to wait for each one to finish. My assumption is that creating the files(disk operation) will take WAY longer than splitting up the data. and of course you should verify first that a sequential approach is not adequate.

What are the alternatives now that the Google web search API has been deprecated?

I have just come across this from Common Crawl.

http://www.commoncrawl.org/

Might be the answer we are all looking for!!

Laravel - Eloquent or Fluent random row

In Laravel 7.x and above, you can just do:

$data = Images::all()->random(4);

Bootstrap 3.0 - Fluid Grid that includes Fixed Column Sizes

Updated 2018

IMO, the best way to approach this in Bootstrap 3 would be using media queries that align with Bootstrap's breakpoints so that you only use the fixed width columns are larger screens and then let the layout stack responsively on smaller screens. This way you keep the responsiveness...

@media (min-width:768px) {
  #sidebar {
      width: inherit;
      min-width: 240px;
      max-width: 240px;
      min-height: 100%;
      position:relative;
  }
  #sidebar2 {
      min-width: 160px;
      max-width: 160px;
      min-height: 100%;
      position:relative;
  }
  #main {
      width:calc(100% - 400px);
  }
}

Working Bootstrap Fixed-Fluid Demo

Bootstrap 4 will has flexbox so layouts like this will be much easier: http://www.codeply.com/go/eAYKvDkiGw

Why is it important to override GetHashCode when Equals method is overridden?

Yes, it is important if your item will be used as a key in a dictionary, or HashSet<T>, etc - since this is used (in the absence of a custom IEqualityComparer<T>) to group items into buckets. If the hash-code for two items does not match, they may never be considered equal (Equals will simply never be called).

The GetHashCode() method should reflect the Equals logic; the rules are:

  • if two things are equal (Equals(...) == true) then they must return the same value for GetHashCode()
  • if the GetHashCode() is equal, it is not necessary for them to be the same; this is a collision, and Equals will be called to see if it is a real equality or not.

In this case, it looks like "return FooId;" is a suitable GetHashCode() implementation. If you are testing multiple properties, it is common to combine them using code like below, to reduce diagonal collisions (i.e. so that new Foo(3,5) has a different hash-code to new Foo(5,3)):

unchecked // only needed if you're compiling with arithmetic checks enabled
{ // (the default compiler behaviour is *disabled*, so most folks won't need this)
    int hash = 13;
    hash = (hash * 7) + field1.GetHashCode();
    hash = (hash * 7) + field2.GetHashCode();
    ...
    return hash;
}

Oh - for convenience, you might also consider providing == and != operators when overriding Equals and GetHashCode.


A demonstration of what happens when you get this wrong is here.

HTML5 tag for horizontal line break

Instead of using <hr>, you can one of the border of the enclosing block and display it as a horizontal line.

Here is a sample code:

The HTML:

<div class="title_block">
    <h3>This is a header.</h3>
</div>
<p>Here is some sample paragraph text.<br>
This demonstrates that a horizontal line goes between the title and the paragraph.</p>

The CSS:

.title_block {
    border-bottom: 1px solid #ddd;
    padding-bottom: 5px;
    margin-bottom: 5px;
}

How to use \n new line in VB msgbox() ...?

Use the command "vbNewLine"

Example

Hello & vbNewLine & "World"

will show up as Hello on one line and World on another

Error: "an object reference is required for the non-static field, method or property..."

Change your signatures to private static bool siprimo(long a) and private static long volteado(long a) and see where that gets you.

How do I capture the output into a variable from an external process in PowerShell?

Or try this. It will capture output into variable $scriptOutput:

& "netdom.exe" $params | Tee-Object -Variable scriptOutput | Out-Null

$scriptOutput

Loading basic HTML in Node.js

I know this is an old question, but as no one has mentioned it I thought it was worth adding:

If you literally want to serve static content (say an 'about' page, image, css, etc) you can use one of the static content serving modules, for example node-static. (There's others that may be better/worse - try search.npmjs.org.) With a little bit of pre-processing you can then filter dynamic pages from static and send them to the right request handler.

What do the python file extensions, .pyc .pyd .pyo stand for?

  1. .py: This is normally the input source code that you've written.
  2. .pyc: This is the compiled bytecode. If you import a module, python will build a *.pyc file that contains the bytecode to make importing it again later easier (and faster).
  3. .pyo: This was a file format used before Python 3.5 for *.pyc files that were created with optimizations (-O) flag. (see the note below)
  4. .pyd: This is basically a windows dll file. http://docs.python.org/faq/windows.html#is-a-pyd-file-the-same-as-a-dll

Also for some further discussion on .pyc vs .pyo, take a look at: http://www.network-theory.co.uk/docs/pytut/CompiledPythonfiles.html (I've copied the important part below)

  • When the Python interpreter is invoked with the -O flag, optimized code is generated and stored in ‘.pyo’ files. The optimizer currently doesn't help much; it only removes assert statements. When -O is used, all bytecode is optimized; .pyc files are ignored and .py files are compiled to optimized bytecode.
  • Passing two -O flags to the Python interpreter (-OO) will cause the bytecode compiler to perform optimizations that could in some rare cases result in malfunctioning programs. Currently only __doc__ strings are removed from the bytecode, resulting in more compact ‘.pyo’ files. Since some programs may rely on having these available, you should only use this option if you know what you're doing.
  • A program doesn't run any faster when it is read from a ‘.pyc’ or ‘.pyo’ file than when it is read from a ‘.py’ file; the only thing that's faster about ‘.pyc’ or ‘.pyo’ files is the speed with which they are loaded.
  • When a script is run by giving its name on the command line, the bytecode for the script is never written to a ‘.pyc’ or ‘.pyo’ file. Thus, the startup time of a script may be reduced by moving most of its code to a module and having a small bootstrap script that imports that module. It is also possible to name a ‘.pyc’ or ‘.pyo’ file directly on the command line.

Note:

On 2015-09-15 the Python 3.5 release implemented PEP-488 and eliminated .pyo files. This means that .pyc files represent both unoptimized and optimized bytecode.

Example of waitpid() in use?

The syntax is

pid_t waitpid(pid_t pid, int *statusPtr, int options);

1.where pid is the process of the child it should wait.

2.statusPtr is a pointer to the location where status information for the terminating process is to be stored.

3.specifies optional actions for the waitpid function. Either of the following option flags may be specified, or they can be combined with a bitwise inclusive OR operator:

WNOHANG WUNTRACED WCONTINUED

If successful, waitpid returns the process ID of the terminated process whose status was reported. If unsuccessful, a -1 is returned.

benifits over wait

1.Waitpid can used when you have more than one child for the process and you want to wait for particular child to get its execution done before parent resumes

2.waitpid supports job control

3.it supports non blocking of the parent process

How to create a delay in Swift?

I agree with Palle that using dispatch_after is a good choice here. But you probably don't like the GCD calls as they are quite annoying to write. Instead you can add this handy helper:

public func delay(bySeconds seconds: Double, dispatchLevel: DispatchLevel = .main, closure: @escaping () -> Void) {
    let dispatchTime = DispatchTime.now() + seconds
    dispatchLevel.dispatchQueue.asyncAfter(deadline: dispatchTime, execute: closure)
}

public enum DispatchLevel {
    case main, userInteractive, userInitiated, utility, background
    var dispatchQueue: DispatchQueue {
        switch self {
        case .main:                 return DispatchQueue.main
        case .userInteractive:      return DispatchQueue.global(qos: .userInteractive)
        case .userInitiated:        return DispatchQueue.global(qos: .userInitiated)
        case .utility:              return DispatchQueue.global(qos: .utility)
        case .background:           return DispatchQueue.global(qos: .background)
        }
    }
}

Now you simply delay your code on a background thread like this:

delay(bySeconds: 1.5, dispatchLevel: .background) { 
    // delayed code that will run on background thread
}

Delaying code on the main thread is even simpler:

delay(bySeconds: 1.5) { 
    // delayed code, by default run in main thread
}

If you prefer a Framework that also has some more handy features then checkout HandySwift. You can add it to your project via SwiftPM then use it exactly like in the examples above:

import HandySwift    

delay(by: .seconds(1.5)) { 
    // delayed code
}

How do I add files and folders into GitHub repos?

When adding a directory to github check that the directory does not contain a .git file using "ls -a" if it does remove it. .git files in a directory will cause problems when you are trying to add a that directory in git

Experimental decorators warning in TypeScript compilation

have to add typescript.tsdk to my .vscode/settings.json:

"typescript.tsdk": "node_modules/typescript/lib"

TypeError: method() takes 1 positional argument but 2 were given

In my case, I forgot to add the ()

I was calling the method like this

obj = className.myMethod

But it should be is like this

obj = className.myMethod()

How to get a random number in Ruby

Well, I figured it out. Apparently there is a builtin (?) function called rand:

rand(n + 1)

If someone answers with a more detailed answer, I'll mark that as the correct answer.

How to convert a list into data table

you can use this extension method and call it like this.

DataTable dt =   YourList.ToDataTable();

public static DataTable ToDataTable<T>(this List<T> iList)
        {
            DataTable dataTable = new DataTable();
            PropertyDescriptorCollection propertyDescriptorCollection =
                TypeDescriptor.GetProperties(typeof(T));
            for (int i = 0; i < propertyDescriptorCollection.Count; i++)
            {
                PropertyDescriptor propertyDescriptor = propertyDescriptorCollection[i];
                Type type = propertyDescriptor.PropertyType;

                if (type.IsGenericType && type.GetGenericTypeDefinition() == typeof(Nullable<>))
                    type = Nullable.GetUnderlyingType(type);


                dataTable.Columns.Add(propertyDescriptor.Name, type);
            }
            object[] values = new object[propertyDescriptorCollection.Count];
            foreach (T iListItem in iList)
            {
                for (int i = 0; i < values.Length; i++)
                {
                    values[i] = propertyDescriptorCollection[i].GetValue(iListItem);
                }
                dataTable.Rows.Add(values);
            }
            return dataTable;
        }

How do I set vertical space between list items?

you can also use the line-height property on the ul

_x000D_
_x000D_
ul {
  line-height: 45px;
}
_x000D_
<ul>
<li>line one</li>
<li>line two</li>
<li>line three</li>
</ul>
_x000D_
_x000D_
_x000D_

Element count of an array in C++

Let's say I have an array arr. When would the following not give the number of elements of the array: sizeof(arr) / sizeof(arr[0])?

One thing I've often seen new programmers doing this:

void f(Sample *arr)
{
   int count = sizeof(arr)/sizeof(arr[0]); //what would be count? 10?
}

Sample arr[10];
f(arr);

So new programmers think the value of count will be 10. But that's wrong.

Even this is wrong:

void g(Sample arr[]) //even more deceptive form!
{
   int count = sizeof(arr)/sizeof(arr[0]); //count would not be 10  
}

It's all because once you pass an array to any of these functions, it becomes pointer type, and so sizeof(arr) would give the size of pointer, not array!


EDIT:

The following is an elegant way you can pass an array to a function, without letting it to decay into pointer type:

template<size_t N>
void h(Sample (&arr)[N])
{
    size_t count = N; //N is 10, so would be count!
    //you can even do this now:
    //size_t count = sizeof(arr)/sizeof(arr[0]);  it'll return 10!
}
Sample arr[10];
h(arr); //pass : same as before!

TypeScript and field initializers

I wanted a solution that would have the following:

  • All the data objects are required and must be filled by the constructor.
  • No need to provide defaults.
  • Can use functions inside the class.

Here is the way that I do it:

export class Person {
  id!: number;
  firstName!: string;
  lastName!: string;

  getFullName() {
    return `${this.firstName} ${this.lastName}`;
  }

  constructor(data: OnlyData<Person>) {
    Object.assign(this, data);
  }
}

const person = new Person({ id: 5, firstName: "John", lastName: "Doe" });
person.getFullName();

All the properties in the constructor are mandatory and may not be omitted without a compiler error.

It is dependant on the OnlyData that filters out getFullName() out of the required properties and it is defined like so:

// based on : https://medium.com/dailyjs/typescript-create-a-condition-based-subset-types-9d902cea5b8c
type FilterFlags<Base, Condition> = { [Key in keyof Base]: Base[Key] extends Condition ? never : Key };
type AllowedNames<Base, Condition> = FilterFlags<Base, Condition>[keyof Base];
type SubType<Base, Condition> = Pick<Base, AllowedNames<Base, Condition>>;
type OnlyData<T> = SubType<T, (_: any) => any>;

Current limitations of this way:

  • Requires TypeScript 2.8
  • Classes with getters/setters

recursion versus iteration

Question :

And if recursion is usually slower what is the technical reason for ever using it over for loop iteration?

Answer :

Because in some algorithms are hard to solve it iteratively. Try to solve depth-first search in both recursively and iteratively. You will get the idea that it is plain hard to solve DFS with iteration.

Another good thing to try out : Try to write Merge sort iteratively. It will take you quite some time.

Question :

Is it correct to say that everywhere recursion is used a for loop could be used?

Answer :

Yes. This thread has a very good answer for this.

Question :

And if it is always possible to convert an recursion into a for loop is there a rule of thumb way to do it?

Answer :

Trust me. Try to write your own version to solve depth-first search iteratively. You will notice that some problems are easier to solve it recursively.

Hint : Recursion is good when you are solving a problem that can be solved by divide and conquer technique.

How to remove stop words using nltk or python

from nltk.corpus import stopwords 

from nltk.tokenize import word_tokenize 

example_sent = "This is a sample sentence, showing off the stop words filtration."

  
stop_words = set(stopwords.words('english')) 
  
word_tokens = word_tokenize(example_sent) 
  
filtered_sentence = [w for w in word_tokens if not w in stop_words] 
  
filtered_sentence = [] 
  
for w in word_tokens: 
    if w not in stop_words: 
        filtered_sentence.append(w) 
  
print(word_tokens) 
print(filtered_sentence) 

Making text bold using attributed string in swift

Improving upon Prajeet Shrestha answer : -

You can make a generic extension for NSMutableAttributedString which involves less code. In this case I have chosen to use system font but you could adapt it so you can input the font name as a parameter.

    extension NSMutableAttributedString {

        func systemFontWith(text: String, size: CGFloat, weight: CGFloat) -> NSMutableAttributedString {
            let attributes: [String: AnyObject] = [NSFontAttributeName: UIFont.systemFont(ofSize: size, weight: weight)]
            let string = NSMutableAttributedString(string: text, attributes: attributes)
            self.append(string)
            return self
        }
    }

How do I install a Python package with a .whl file?

I would be suggesting you the exact way how to install .whl file. Initially I faced many issues but then I solved it, Here is my trick to install .whl files.

Follow The Steps properly in order to get a module imported

  1. Make sure your .whl file is kept in the python 2.7/3.6/3.7/.. folder. Initially when you download the .whl file the file is kept in downloaded folder, my suggestion is to change the folder. It makes it easier to install the file.
  2. Open command prompt and open the folder where you have kept the file by entering

cd c:\python 3.7

3.Now, enter the command written below

>py -3.7(version name) -m pip install (file name).whl
  1. Click enter and make sure you enter the version you are currently using with correct file name.

  2. Once you press enter, wait for few minutes and the file will be installed and you will be able to import the particular module.

  3. In order to check if the module is installed successfully, import the module in idle and check it.

Thank you:)

Reading and writing binary file

Here is a short example, the C++ way using rdbuf. I got this from the web. I can't find my original source on this:

#include <fstream>
#include <iostream>

int main () 
{
  std::ifstream f1 ("C:\\me.txt",std::fstream::binary);

  std::ofstream f2 ("C:\\me2.doc",std::fstream::trunc|std::fstream::binary);

  f2<<f1.rdbuf();

  return 0;
}

How to hide a navigation bar from first ViewController in Swift?

Ways to show Navigation Bar in Swift:

self.navigationController?.setNavigationBarHidden(false, animated: true)
self.navigationController?.navigationBar.isHidden = false
self.navigationController?.isNavigationBarHidden = false

How to count the frequency of the elements in an unordered list?

In Python 2.7 (or newer), you can use collections.Counter:

import collections
a = [1,1,1,1,2,2,2,2,3,3,4,5,5]
counter=collections.Counter(a)
print(counter)
# Counter({1: 4, 2: 4, 3: 2, 5: 2, 4: 1})
print(counter.values())
# [4, 4, 2, 1, 2]
print(counter.keys())
# [1, 2, 3, 4, 5]
print(counter.most_common(3))
# [(1, 4), (2, 4), (3, 2)]

If you are using Python 2.6 or older, you can download it here.

What is CMake equivalent of 'configure --prefix=DIR && make all install '?

Note that in both CMake and Autotools you don't always have to set the installation path at configure time. You can use DESTDIR at install time (see also here) instead as in:

make DESTDIR=<installhere> install

See also this question which explains the subtle difference between DESTDIR and PREFIX.

This is intended for staged installs and to allow for storing programs in a different location from where they are run e.g. /etc/alternatives via symbolic links.

However, if your package is relocatable and doesn't need any hard-coded (prefix) paths set via the configure stage you may be able to skip it. So instead of:

cmake -DCMAKE_INSTALL_PREFIX=/usr . && make all install

you would run:

cmake . && make DESTDIR=/usr all install

Note that, as user7498341 points out, this is not appropriate for cases where you really should be using PREFIX.

How to use org.apache.commons package?

Download commons-net binary from here. Extract the files and reference the commons-net-x.x.jar file.

How do I address unchecked cast warnings?

You can create a utility class like the following, and use it to suppress the unchecked warning.

public class Objects {

    /**
     * Helps to avoid using {@code @SuppressWarnings({"unchecked"})} when casting to a generic type.
     */
    @SuppressWarnings({"unchecked"})
    public static <T> T uncheckedCast(Object obj) {
        return (T) obj;
    }
}

You can use it as follows:

import static Objects.uncheckedCast;
...

HashMap<String, String> getItems(javax.servlet.http.HttpSession session) {
      return uncheckedCast(session.getAttribute("attributeKey"));
}

Some more discussion about this is here: http://cleveralias.blogs.com/thought_spearmints/2006/01/suppresswarning.html

How to remove entry from $PATH on mac

Check the following files:

/etc/bashrc
/etc/profile
~/.bashrc
~/.bash_profile
~/.profile
~/.MacOSX/environment.plist

Some of these files may not exist, but they're the most likely ones to contain $PATH definitions.

Saving numpy array to txt file row wise

Very very easy: [1,2,3]

A list is like a column.

1
2
3

If you want a list like a row, double corchete:

[[1, 2, 3]]  --->    1, 2, 3

and

[[1, 2, 3], [4, 5, 6]]  ---> 1, 2, 3
                             4, 5, 6

Finally:

np.savetxt("file", [['r1c1', 'r1c2'], ['r2c1', 'r2c2']], delimiter=';', fmt='%s')

Note, the comma between square brackets, inner list are elements of the outer list

How to safely call an async method in C# without await

The solution is start the HttpClient into another execution task without sincronization context:

var submit = httpClient.PostAsync(uri, new StringContent(body, Encoding.UTF8,"application/json"));
var t = Task.Run(() => submit.ConfigureAwait(false));
await t.ConfigureAwait(false);

Sleep function in C++

Prior to C++11, there was no portable way to do this.

A portable way is to use Boost or Ace library. There is ACE_OS::sleep(); in ACE.

XMLHttpRequest cannot load file. Cross origin requests are only supported for HTTP

Depends on your needs, but there is also a quick way to temporarily check your (dummy) JSON by saving your JSON on http://myjson.com. Copy the api link and paste that into your javascript code. Viola! When you want to deploy the codes, you must not forget to change that url in your codes!

The entity cannot be constructed in a LINQ to Entities query

In response to the other question which was marked as duplicate (see here) I figured out a quick and easy solution based on the answer of Soren:

data.Tasks.AddRange(
    data.Task.AsEnumerable().Select(t => new Task{
        creator_id   = t.ID,
        start_date   = t.Incident.DateOpened,
        end_date     = t.Incident.DateCLosed,
        product_code = t.Incident.ProductCode
        // so on...
    })
);
data.SaveChanges();

Note: This solution only works if you have a navigation property (foreign key) on the Task class (here called 'Incident'). If you don't have that, you can just use one of the other posted solutions with "AsQueryable()".

Run-time error '3061'. Too few parameters. Expected 1. (Access 2007)

Thanks for John Doe's solution that helped a lot. Mine is very similar with some difference, using TempVars

Instead of :

      strSQL = "SELECT * FROM Table1 WHERE Name = Forms!Table!Control;"

I used:

      strSQL = "SELECT * FROM Query1"  , Query1 being common for other usage

Query1 as:

 "Select Field1, Field2 from Table1 where Id= [TempVars]![MyVar]

And similarly, removing [TempVars]![MyVar] from view solved the problem.

Find directory name with wildcard or similar to "like"

find supports wildcard matches, just add a *:

find / -type d -name "ora10*"

How many bytes is unsigned long long?

Executive summary: it's 64 bits, or larger.

unsigned long long is the same as unsigned long long int. Its size is platform-dependent, but guaranteed by the C standard (ISO C99) to be at least 64 bits. There was no long long in C89, but apparently even MSVC supports it, so it's quite portable.

In the current C++ standard (issued in 2003), there is no long long, though many compilers support it as an extension. The upcoming C++0x standard will support it and its size will be the same as in C, so at least 64 bits.

You can get the exact size, in bytes (8 bits on typical platforms) with the expression sizeof(unsigned long long). If you want exactly 64 bits, use uint64_t, which is defined in the header <stdint.h> along with a bunch of related types (available in C99, C++11 and some current C++ compilers).

What does DIM stand for in Visual Basic and BASIC?

Dim originally (in BASIC) stood for Dimension, as it was used to define the dimensions of an array.

(The original implementation of BASIC was Dartmouth BASIC, which descended from FORTRAN, where DIMENSION is spelled out.)

Nowadays, Dim is used to define any variable, not just arrays, so its meaning is not intuitive anymore.

Laravel 5 Eloquent where and or in Clauses

You can try to use the following code instead:

 $pro= model_name::where('col_name', '=', 'value')->get();

How to set the min and max height or width of a Frame?

A workaround - at least for the minimum size: You can use grid to manage the frames contained in root and make them follow the grid size by setting sticky='nsew'. Then you can use root.grid_rowconfigure and root.grid_columnconfigure to set values for minsize like so:

from tkinter import Frame, Tk

class MyApp():
    def __init__(self):
        self.root = Tk()

        self.my_frame_red = Frame(self.root, bg='red')
        self.my_frame_red.grid(row=0, column=0, sticky='nsew')

        self.my_frame_blue = Frame(self.root, bg='blue')
        self.my_frame_blue.grid(row=0, column=1, sticky='nsew')

        self.root.grid_rowconfigure(0, minsize=200, weight=1)
        self.root.grid_columnconfigure(0, minsize=200, weight=1)
        self.root.grid_columnconfigure(1, weight=1)

        self.root.mainloop()

if __name__ == '__main__':
    app = MyApp()

But as Brian wrote (in 2010 :D) you can still resize the window to be smaller than the frame if you don't limit its minsize.

JAXB Exception: Class not known to this context

I had this error because I registered the wrong class in this line of code:

JAXBContext context = JAXBContext.newInstance(MyRootXmlClass.class);

SET NOCOUNT ON usage

Regarding the triggers breaking NHibernate, I had that experience first-hand. Basically, when NH does an UPDATE it expects certain number of rows affected. By adding SET NOCOUNT ON to the triggers you get the number of rows back to what NH expected thereby fixing the issue. So yeah, I would definitely recommend turning it off for triggers if you use NH.

Regarding the usage in SPs, it's a matter of personal preference. I had always turned the row count off, but then again, there are no real strong arguments either way.

On a different note, you should really consider moving away from SP-based architecture, then you won't even have this question.

Bootstrap datepicker disabling past dates without current date

You can find your solution in this link below: https://codepen.io/ahmetcadirci25/pen/NpMNzJ

Thats work for me.

My code:

        var date = new Date();
        date.setDate(date.getDate());

        $('#datetimepicker1').datetimepicker({
            isRTL: false,
            format: 'dd.mm.yyyy hh:ii',
            autoclose: true,
            language: 'tr',
            startDate: date
        });

Get text of label with jquery

Try:

<%=this.Label1.Text%>

How do I resolve this "ORA-01109: database not open" error?

If your database is down then during login as SYSDBA you can assume this. While login command will be executed like sqlplus sys/sys_password as sysdba that time you will get "connected to idle instance" reply from database. This message indicates your database is down. You should need to check first alert.log file about why database is down. If you found it was downed normally then you can issue "startup" command for starting database and after that execute your create user command. If you found database is having issue like missing datafile or something else then you need to recover database first and open database for executing your create user command.

"alter database open" command only accepted by database while it is on Mount stage. If database is down then it won't accept "alter database open" command.

ORA-01008: not all variables bound. They are bound

I'd a similar problem in a legacy application, but de "--" was string parameter.

Ex.:

Dim cmd As New OracleCommand("INSERT INTO USER (name, address, photo) VALUES ('User1', '--', :photo)", oracleConnection)
Dim fs As IO.FileStream = New IO.FileStream("c:\img.jpg", IO.FileMode.Open)
Dim br As New IO.BinaryReader(fs)
cmd.Parameters.Add(New OracleParameter("photo", OracleDbType.Blob)).Value = br.ReadBytes(fs.Length)
cmd.ExecuteNonQuery() 'here throws ORA-01008

Changing address parameter value '--' to '00' or other thing, works.

Passing parameter to controller action from a Html.ActionLink

Addition to the accepted answer:

if you are going to use

 @Html.ActionLink("LinkName", "ActionName", "ControllerName", new { @id = idValue, @secondParam= = 2 },null)

this will create actionlink where you can't create new custom attribute or style for the link.

However, the 4th parameter in ActionLink extension will solve that problem. Use the 4th parameter for customization in your way.

 @Html.ActionLink("LinkName", "ActionName", "ControllerName", new { @id = idValue, @secondParam= = 2 }, new { @class = "btn btn-info", @target = "_blank" })

Get first element from a dictionary

For anyone coming to this that wants a linq-less way to get an element from a dictionary

var d = new Dictionary<string, string>();
d.Add("a", "b");
var e = d.GetEnumerator();
e.MoveNext();
var anElement = e.Current;
// anElement/e.Current is a KeyValuePair<string,string>
// where Key = "a", Value = "b"

I'm not sure if this is implementation specific, but if your Dictionary doesn't have any elements, Current will contain a KeyValuePair<string, string> where both the key and value are null.

(I looked at the logic behind linq's First method to come up with this, and tested it via LinqPad 4)

How to parse XML using vba

This is a bit of a complicated question, but it seems like the most direct route would be to load the XML document or XML string via MSXML2.DOMDocument which will then allow you to access the XML nodes.

You can find more on MSXML2.DOMDocument at the following sites:

How to convert int to Integer

I had a similar problem . For this you can use a Hashmap which takes "string" and "object" as shown in code below:

/** stores the image database icons */
public static int[] imageIconDatabase = { R.drawable.ball,
        R.drawable.catmouse, R.drawable.cube, R.drawable.fresh,
        R.drawable.guitar, R.drawable.orange, R.drawable.teapot,
        R.drawable.india, R.drawable.thailand, R.drawable.netherlands,
        R.drawable.srilanka, R.drawable.pakistan,

};
private void initializeImageList() {
    // TODO Auto-generated method stub
    for (int i = 0; i < imageIconDatabase.length; i++) {
        map = new HashMap<String, Object>();

        map.put("Name", imageNameDatabase[i]);
        map.put("Icon", imageIconDatabase[i]);
    }

}

How to automatically allow blocked content in IE?

There is a code solution too. I saw it in a training video. You can add a line to tell IE that the local file is safe. I tested on IE8 and it works. That line is <!-- saved from url=(0014)about:internet -->

For more details, please refer to https://msdn.microsoft.com/en-us/library/ms537628(v=vs.85).aspx

<!DOCTYPE html>
<!-- saved from url=(0014)about:internet -->
<html lang="en">
    <title></title>
    <script type="text/javascript" src="http://code.jquery.com/jquery-latest.js"></script>
    <script>
        $(document).ready(function () {
            alert('hi');

        });
    </script>
</head>
<body>
</body>
</html>

Get hostname of current request in node.js Express

If you need a fully qualified domain name and have no HTTP request, on Linux, you could use:

var child_process = require("child_process");

child_process.exec("hostname -f", function(err, stdout, stderr) {
  var hostname = stdout.trim();
});

Add borders to cells in POI generated Excel File

If you're using the org.apache.poi.ss.usermodel (not HSSF or XSSF) you can use:

style.setBorderBottom(BorderStyle.THIN);
style.setBorderTop(BorderStyle.THIN);
style.setBorderLeft(BorderStyle.THIN);
style.setBorderRight(BorderStyle.THIN);

all the border styles are here at the apache documentation

How to run script as another user without password?

Call visudo and add this:

user1 ALL=(user2) NOPASSWD: /home/user2/bin/test.sh

The command paths must be absolute! Then call sudo -u user2 /home/user2/bin/test.sh from a user1 shell. Done.

Webdriver findElements By xpath

The XPath turns into this:

Get me all of the div elements that have an id equal to container.

As for getting the first etc, you have two options.

Turn it into a .findElement() - this will just return the first one for you anyway.

or

To explicitly do this in XPath, you'd be looking at:

(//div[@id='container'])[1]

for the first one, for the second etc:

(//div[@id='container'])[2]

Then XPath has a special indexer, called last, which would (you guessed it) get you the last element found:

(//div[@id='container'])[last()]

Worth mentioning that XPath indexers will start from 1 not 0 like they do in most programming languages.

As for getting the parent 'node', well, you can use parent:

//div[@id='container']/parent::*

That would get the div's direct parent.

You could then go further and say I want the first *div* with an id of container, and I want his parent:

(//div[@id='container'])[1]/parent::*

Hope that helps!

How do I restrict an input to only accept numbers?

DECIMAL

directive('decimal', function() {
                return {
                    require: 'ngModel',
                    restrict: 'A',
                    link: function(scope, element, attr, ctrl) {
                        function inputValue(val) {
                            if (val) {
                                var digits = val.replace(/[^0-9.]/g, '');

                                if (digits.split('.').length > 2) {
                                    digits = digits.substring(0, digits.length - 1);
                                }

                                if (digits !== val) {
                                    ctrl.$setViewValue(digits);
                                    ctrl.$render();
                                }
                                return parseFloat(digits);
                            }
                            return "";
                        }
                        ctrl.$parsers.push(inputValue);
                    }
                };
            });

DIGITS

directive('entero', function() {
            return {
                require: 'ngModel',
                restrict: 'A',
                link: function(scope, element, attr, ctrl) {
                    function inputValue(val) {
                        if (val) {
                            var value = val + ''; //convert to string
                            var digits = value.replace(/[^0-9]/g, '');

                            if (digits !== value) {
                                ctrl.$setViewValue(digits);
                                ctrl.$render();
                            }
                            return parseInt(digits);
                        }
                        return "";
                    }
                    ctrl.$parsers.push(inputValue);
                }
            };
        });

angular directives for validate numbers

How to format number of decimal places in wpf using style/template?

You should use the StringFormat on the Binding. You can use either standard string formats, or custom string formats:

<TextBox Text="{Binding Value, StringFormat=N2}" />
<TextBox Text="{Binding Value, StringFormat={}{0:#,#.00}}" />

Note that the StringFormat only works when the target property is of type string. If you are trying to set something like a Content property (typeof(object)), you will need to use a custom StringFormatConverter (like here), and pass your format string as the ConverterParameter.

Edit for updated question

So, if your ViewModel defines the precision, I'd recommend doing this as a MultiBinding, and creating your own IMultiValueConverter. This is pretty annoying in practice, to go from a simple binding to one that needs to be expanded out to a MultiBinding, but if the precision isn't known at compile time, this is pretty much all you can do. Your IMultiValueConverter would need to take the value, and the precision, and output the formatted string. You'd be able to do this using String.Format.

However, for things like a ContentControl, you can much more easily do this with a Style:

<Style TargetType="{x:Type ContentControl}">
    <Setter Property="ContentStringFormat" 
            Value="{Binding Resolution, StringFormat=N{0}}" />
</Style>

Any control that exposes a ContentStringFormat can be used like this. Unfortunately, TextBox doesn't have anything like that.

Relative paths in Python

Consider my code:

import os


def readFile(filename):
    filehandle = open(filename)
    print filehandle.read()
    filehandle.close()



fileDir = os.path.dirname(os.path.realpath('__file__'))
print fileDir

#For accessing the file in the same folder
filename = "same.txt"
readFile(filename)

#For accessing the file in a folder contained in the current folder
filename = os.path.join(fileDir, 'Folder1.1/same.txt')
readFile(filename)

#For accessing the file in the parent folder of the current folder
filename = os.path.join(fileDir, '../same.txt')
readFile(filename)

#For accessing the file inside a sibling folder.
filename = os.path.join(fileDir, '../Folder2/same.txt')
filename = os.path.abspath(os.path.realpath(filename))
print filename
readFile(filename)

How to concatenate strings in django templates?

You can't do variable manipulation in django templates. You have two options, either write your own template tag or do this in view,

How to enable native resolution for apps on iPhone 6 and 6 Plus?

Do the following (see in photo)

  1. Goto asset catalog
  2. right-click and choose "Add New Launch Image"

    • iPhone 6 -> 750 x 1334
    • iPhone 6 Plus -> 1242 x 2208 and 2208 x 1242

enter image description here

What's the difference between '$(this)' and 'this'?

$() is the jQuery constructor function.

this is a reference to the DOM element of invocation.

So basically, in $(this), you are just passing the this in $() as a parameter so that you could call jQuery methods and functions.

Storing C++ template function definitions in a .CPP file

Time for an update! Create an inline (.inl, or probably any other) file and simply copy all your definitions in it. Be sure to add the template above each function (template <typename T, ...>). Now instead of including the header file in the inline file you do the opposite. Include the inline file after the declaration of your class (#include "file.inl").

I don't really know why no one has mentioned this. I see no immediate drawbacks.

Use of Finalize/Dispose method in C#

Pattern from msdn

public class BaseResource: IDisposable
{
   private IntPtr handle;
   private Component Components;
   private bool disposed = false;
   public BaseResource()
   {
   }
   public void Dispose()
   {
      Dispose(true);      
      GC.SuppressFinalize(this);
   }
   protected virtual void Dispose(bool disposing)
   {
      if(!this.disposed)
      {        
         if(disposing)
         {
            Components.Dispose();
         }         
         CloseHandle(handle);
         handle = IntPtr.Zero;
       }
      disposed = true;         
   }
   ~BaseResource()      
   {      Dispose(false);
   }
   public void DoSomething()
   {
      if(this.disposed)
      {
         throw new ObjectDisposedException();
      }
   }
}
public class MyResourceWrapper: BaseResource
{
   private ManagedResource addedManaged;
   private NativeResource addedNative;
   private bool disposed = false;
   public MyResourceWrapper()
   {
   }
   protected override void Dispose(bool disposing)
   {
      if(!this.disposed)
      {
         try
         {
            if(disposing)
            {             
               addedManaged.Dispose();         
            }
            CloseHandle(addedNative);
            this.disposed = true;
         }
         finally
         {
            base.Dispose(disposing);
         }
      }
   }
}

How to decrypt an encrypted Apple iTunes iPhone backup?

Sorry, but it might even be more complicated, involving pbkdf2, or even a variation of it. Listen to the WWDC 2010 session #209, which mainly talks about the security measures in iOS 4, but also mentions briefly the separate encryption of backups and how they're related.

You can be pretty sure that without knowing the password, there's no way you can decrypt it, even by brute force.

Let's just assume you want to try to enable people who KNOW the password to get to the data of their backups.

I fear there's no way around looking at the actual code in iTunes in order to figure out which algos are employed.

Back in the Newton days, I had to decrypt data from a program and was able to call its decryption function directly (knowing the password, of course) without the need to even undersand its algorithm. It's not that easy anymore, unfortunately.

I'm sure there are skilled people around who could reverse engineer that iTunes code - you just have to get them interested.

In theory, Apple's algos should be designed in a way that makes the data still safe (i.e. practically unbreakable by brute force methods) to any attacker knowing the exact encryption method. And in WWDC session 209 they went pretty deep into details about what they do to accomplish this. Maybe you can actually get answers directly from Apple's security team if you tell them your good intentions. After all, even they should know that security by obfuscation is not really efficient. Try their security mailing list. Even if they do not repond, maybe someone else silently on the list will respond with some help.

Good luck!

Postgresql tables exists, but getting "relation does not exist" when querying

I was using pgAdmin to create my tables and while I was not using reserved words, the generated table had a quote in the name and a couple of columns had quotes in them. Here is an example of the generated SQL.

CREATE TABLE public."Test"
(
    id serial NOT NULL,
    data text NOT NULL,
    updater character varying(50) NOT NULL,
    "updateDt" time with time zone NOT NULL,
    CONSTRAINT test_pk PRIMARY KEY (id)
)

TABLESPACE pg_default;

ALTER TABLE public."Test"
    OWNER to svc_newnews_app;

All of these quotes were inserted at "random". I just needed to drop and re-create the table again without the quotes.

Tested on pgAdmin 4.26

Why does "pip install" inside Python raise a SyntaxError?

Programmatically, the following currently works. I see all the answers post 10.0 and all, but none of them are the correct path for me. Within Kaggle for sure, this apporach works

from pip._internal import main as _main

package_names=['pandas'] #packages to install
_main(['install'] + package_names + ['--upgrade']) 

Deleting records before a certain date

DELETE FROM table WHERE date < '2011-09-21 08:21:22';

Adding elements to a C# array

You can use a generic collection, like List<>

List<string> list = new List<string>();

// add
list.Add("element");

// remove
list.Remove("element");

Is there a way to run Python on Android?

Didn't see this posted here, but you can do it with Pyside and Qt now that Qt works on Android thanks to Necessitas.

It seems like quite a kludge at the moment but could be a viable route eventually...

http://qt-project.org/wiki/PySide_for_Android_guide

PHP using Gettext inside <<<EOF string

As far as I can see in the manual, it is not possible to call functions inside HEREDOC strings. A cumbersome way would be to prepare the words beforehand:

<?php

    $world = _("World");

    $str = <<<EOF
    <p>Hello</p>
    <p>$world</p>
EOF;
    echo $str;
?>

a workaround idea that comes to mind is building a class with a magic getter method.

You would declare a class like this:

class Translator
{
 public function __get($name) {
  return _($name); // Does the gettext lookup
  }
 }

Initialize an object of the class at some point:

  $translate = new Translator();

You can then use the following syntax to do a gettext lookup inside a HEREDOC block:

    $str = <<<EOF
    <p>Hello</p>
    <p>{$translate->World}</p>
EOF;
    echo $str;
?>

$translate->World will automatically be translated to the gettext lookup thanks to the magic getter method.

To use this method for words with spaces or special characters (e.g. a gettext entry named Hello World!!!!!!, you will have to use the following notation:

 $translate->{"Hello World!!!!!!"}

This is all untested but should work.

Update: As @mario found out, it is possible to call functions from HEREDOC strings after all. I think using getters like this is a sleek solution, but using a direct function call may be easier. See the comments on how to do this.

How to use a Java8 lambda to sort a stream in reverse order?

If your stream elements implements Comparable then the solution becomes simpler:

 ...stream()
 .sorted(Comparator.reverseOrder())

Why is __init__() always called after __new__()?

One should look at __init__ as a simple constructor in traditional OO languages. For example, if you are familiar with Java or C++, the constructor is passed a pointer to its own instance implicitly. In the case of Java, it is the this variable. If one were to inspect the byte code generated for Java, one would notice two calls. The first call is to an "new" method, and then next call is to the init method (which is the actual call to the user defined constructor). This two step process enables creation of the actual instance before calling the constructor method of the class which is just another method of that instance.

Now, in the case of Python, __new__ is a added facility that is accessible to the user. Java does not provide that flexibility, due to its typed nature. If a language provided that facility, then the implementor of __new__ could do many things in that method before returning the instance, including creating a totally new instance of a unrelated object in some cases. And, this approach also works out well for especially for immutable types in the case of Python.

How can I make all images of different height and width the same via CSS?

You can use the object-fit property to size the img elements:

  • cover stretches or shrinks the image proportionally to fill the container. The image is cropped horizontally -or- vertically if necessary.
  • contain stretches or shrinks the image proportionally to fit inside the container.
  • scale-down shrinks the image proportionally to fit inside the container.

_x000D_
_x000D_
.example {_x000D_
  margin: 1em 0;_x000D_
  text-align: center;_x000D_
}_x000D_
_x000D_
.example img {_x000D_
  width: 30vw;_x000D_
  height: 30vw;_x000D_
}_x000D_
_x000D_
.example-cover img {_x000D_
  object-fit: cover;_x000D_
}_x000D_
_x000D_
.example-contain img {_x000D_
  object-fit: contain;_x000D_
}
_x000D_
<div class="example example-cover">_x000D_
  <img src="https://i.stack.imgur.com/B0EAo.png">_x000D_
  <img src="https://i.stack.imgur.com/iYkNH.png">_x000D_
  <img src="https://i.stack.imgur.com/gne9N.png">_x000D_
</div>_x000D_
_x000D_
<div class="example example-contain">_x000D_
  <img src="https://i.stack.imgur.com/B0EAo.png">_x000D_
  <img src="https://i.stack.imgur.com/iYkNH.png">_x000D_
  <img src="https://i.stack.imgur.com/gne9N.png">_x000D_
</div>
_x000D_
_x000D_
_x000D_

In the above example: red is landscape, green is portrait and blue is square image. The checkered pattern consists of 16x16px squares.

Create Windows service from executable

To create a Windows Service from an executable, you can use sc.exe:

sc.exe create <new_service_name> binPath= "<path_to_the_service_executable>"

You must have quotation marks around the actual exe path, and a space after the binPath=.

More information on the sc command can be found in Microsoft KB251192.

Note that it will not work for just any executable: the executable must be a Windows Service (i.e. implement ServiceMain). When registering a non-service executable as a service, you'll get the following error upon trying to start the service:

Error 1053: The service did not respond to the start or control request in a timely fashion.

There are tools that can create a Windows Service from arbitrary, non-service executables, see the other answers for examples of such tools.

Protractor : How to wait for page complete after click a button?

to wait until the click itself is complete (ie to resolve the Promise), use await keyword

it('test case 1', async () => {
  await login.submit.click();
})

This will stop the command queue until the click (sendKeys, sleep or any other command) is finished

If you're lucky and you're on angular page that is built well and doesn't have micro and macro tasks pending then Protractor should wait by itself until the page is ready. But sometimes you need to handle waiting yourself, for example when logging in through a page that is not Angular (read how to find out if page has pending tasks and how to work with non angular pages)

In the case you're handling the waiting manually, browser.wait is the way to go. Just pass a function to it that would have a condition which to wait for. For example wait until there is no loading animation on the page

let $animation = $$('.loading');

await browser.wait(
  async () => (await animation.count()) === 0, // function; if returns true it stops waiting; can wait for anything in the world if you get creative with it
  5000, // timeout
  `message on timeout`
);

Make sure to use await

GROUP_CONCAT ORDER BY

Try

SELECT li.clientid, group_concat(li.views ORDER BY li.views) AS views,
       group_concat(li.percentage ORDER BY li.percentage) 
FROM table_views li 
GROUP BY client_id

http://dev.mysql.com/doc/refman/5.0/en/group-by-functions.html#function%5Fgroup-concat

Reading data from XML

Alternatively, you can use XPathNavigator:

XmlDocument doc = new XmlDocument();
doc.LoadXml(xml);
XPathNavigator navigator = doc.CreateNavigator();

string books = GetStringValues("Books: ", navigator, "//Book/Title");
string authors = GetStringValues("Authors: ", navigator, "//Book/Author");

..

/// <summary>
/// Gets the string values.
/// </summary>
/// <param name="description">The description.</param>
/// <param name="navigator">The navigator.</param>
/// <param name="xpath">The xpath.</param>
/// <returns></returns>
private static string GetStringValues(string description,
                                      XPathNavigator navigator, string xpath) {
    StringBuilder sb = new StringBuilder();
    sb.Append(description);
    XPathNodeIterator bookNodesIterator = navigator.Select(xpath);
    while (bookNodesIterator.MoveNext())
       sb.Append(string.Format("{0} ", bookNodesIterator.Current.Value));
    return sb.ToString();
}

Detecting locked tables (locked by LOCK TABLE)

Use SHOW OPEN TABLES: http://dev.mysql.com/doc/refman/5.1/en/show-open-tables.html

You can do something like this

SHOW OPEN TABLES WHERE `Table` LIKE '%[TABLE_NAME]%' AND `Database` LIKE '[DBNAME]' AND In_use > 0;

to check any locked tables in a database.

Convert HTML + CSS to PDF

1) use MPDF !

a) extract in yourfolder

b) create file.php in yourfolder and insert such code:

<?php
include('../mpdf.php');
$mpdf=new mPDF();
$mpdf->WriteHTML('<p style="color:red;">Hallo World<br/>Fisrt sentencee</p>');
$mpdf->Output();   exit;
 ?>

c) open file.php from your browser




2) Use pdfToHtml !

1) extract pdftohtml.exe to your root folder:

2) inside that folder, in anyfile.php file, put this code (assuming, there is a source example.pdf too):

<?php
$source="example.pdf";
$output_fold="FinalFolder";

    if (!file_exists($output_fold)) { mkdir($output_fold, 0777, true);}
$result= passthru("pdftohtml $source $output_fold/new_filename",$log);
//var_dump($result); var_dump($log);
?>

3) enter FinalFolder, and there will be the converted files (as many pages, as the source PDF had..)

How to give a time delay of less than one second in excel vba?

Everyone tries Application.Wait, but that's not really reliable. If you ask it to wait for less than a second, you'll get anything between 0 and 1, but closer to 10 seconds. Here's a demonstration using a wait of 0.5 seconds:

Sub TestWait()
  Dim i As Long
  For i = 1 To 5
    Dim t As Double
    t = Timer
    Application.Wait Now + TimeValue("0:00:00") / 2
    Debug.Print Timer - t
  Next
End Sub

Here's the output, an average of 0.0015625 seconds:

0
0
0
0.0078125
0

Admittedly, Timer may not be the ideal way to measure these events, but you get the idea.

The Timer approach is better:

Sub TestTimer()
  Dim i As Long
  For i = 1 To 5
    Dim t As Double
    t = Timer
    Do Until Timer - t >= 0.5
      DoEvents
    Loop
    Debug.Print Timer - t
  Next
End Sub

And the results average is very close to 0.5 seconds:

0.5
0.5
0.5
0.5
0.5

Read properties file outside JAR file

I have a similar case: wanting my *.jar file to access a file in a directory next to said *.jar file. Refer to THIS ANSWER as well.

My file structure is:

./ - the root of your program
|__ *.jar
|__ dir-next-to-jar/some.txt

I'm able to load a file (say, some.txt) to an InputStream inside the *.jar file with the following:

InputStream stream = null;
    try{
        stream = ThisClassName.class.getClass().getResourceAsStream("/dir-next-to-jar/some.txt");
    }
    catch(Exception e) {
        System.out.print("error file to stream: ");
        System.out.println(e.getMessage());
    }

Then do whatever you will with the stream

Most recent previous business day in Python

Use pandas!

import datetime
# BDay is business day, not birthday...
from pandas.tseries.offsets import BDay

today = datetime.datetime.today()
print(today - BDay(4))

Since today is Thursday, Sept 26, that will give you an output of:

datetime.datetime(2013, 9, 20, 14, 8, 4, 89761)

SQL Format as of Round off removing decimals

CAST(Round(MySum * 20.0 /100, 0) AS INT)

FYI

MySum * 20 /100, I get 11

This is because when all 3 operands are INTs, SQL Server will do perform integer maths, from left to right, truncating all intermediate results.

58 * 20 / 100 => 1160 / 100 => 11 (truncated from 11.6)

Also for the record ROUND(m,n) returns the result to n decimal places, not n significant figures.

Multi-gradient shapes

Try this method then you can do every thing you want.
It is like a stack so be careful which item comes first or last.

<?xml version="1.0" encoding="utf-8"?>
<layer-list
xmlns:android="http://schemas.android.com/apk/res/android">
<item android:right="50dp" android:start="10dp" android:left="10dp">
    <shape
        xmlns:android="http://schemas.android.com/apk/res/android"
        android:shape="rectangle">
        <corners android:radius="3dp" />
        <solid android:color="#012d08"/>
    </shape>
</item>
<item android:top="50dp">
    <shape android:shape="rectangle">
        <solid android:color="#7c4b4b" />
    </shape>
</item>
<item android:top="90dp" android:end="60dp">
    <shape android:shape="rectangle">
        <solid android:color="#e2cc2626" />
    </shape>
</item>
<item android:start="50dp" android:bottom="20dp" android:top="120dp">
    <shape android:shape="rectangle">
        <solid android:color="#360e0e" />
    </shape>
</item>

Is it possible to get multiple values from a subquery?

It's incorrect, but you can try this instead:

select
    a.x,
    ( select b.y from b where b.v = a.v) as by,
    ( select b.z from b where b.v = a.v) as bz
from a

you can also use subquery in join

 select
        a.x,
        b.y,
        b.z
    from a
    left join (select y,z from b where ... ) b on b.v = a.v

or

   select
        a.x,
        b.y,
        b.z
    from a
    left join b on b.v = a.v

How to change btn color in Bootstrap

Remove the button color class like "btn-success" and put a custom class like "btn-custom" and write css for that class. That simply works for me.

HTML :

<button class="btn btn-block login " type="submit">Sign In</button>

CSS:

 .login  {
    background-color: #0057fc;
    color: white;   
}

Return 0 if field is null in MySQL

Yes IFNULL function will be working to achieve your desired result.

SELECT uo.order_id, uo.order_total, uo.order_status,
        (SELECT IFNULL(SUM(uop.price * uop.qty),0) 
         FROM uc_order_products uop 
         WHERE uo.order_id = uop.order_id
        ) AS products_subtotal,
        (SELECT IFNULL(SUM(upr.amount),0) 
         FROM uc_payment_receipts upr 
         WHERE uo.order_id = upr.order_id
        ) AS payment_received,
        (SELECT IFNULL(SUM(uoli.amount),0) 
         FROM uc_order_line_items uoli 
         WHERE uo.order_id = uoli.order_id
        ) AS line_item_subtotal
        FROM uc_orders uo
        WHERE uo.order_status NOT IN ("future", "canceled")
        AND uo.uid = 4172;

Error in your SQL syntax; check the manual that corresponds to your MySQL server version

Use ` backticks for MYSQL reserved words...

table name "table" is reserved word for MYSQL...

so your query should be as follows...

$sql="INSERT INTO `table` (`username`, `password`)
VALUES
('$_POST[username]','$_POST[password]')";

MySQL Error #1133 - Can't find any matching row in the user table

I think the answer is here now : https://bugs.mysql.com/bug.php?id=83822

So, you should write :

GRANT ALL PRIVILEGES ON mydb.* to myuser@'xxx.xxx.xxx.xxx' IDENTIFIED BY 'mypassword';

And i think that could be work :

SET PASSWORD FOR myuser@'xxx.xxx.xxx.xxx' IDENTIFIED BY 'old_password' = PASSWORD('new_password');

C# RSA encryption/decryption with transmission

well there are really enough examples for this, but anyway, here you go

using System;
using System.Security.Cryptography;

namespace RsaCryptoExample
{
  static class Program
  {
    static void Main()
    {
      //lets take a new CSP with a new 2048 bit rsa key pair
      var csp = new RSACryptoServiceProvider(2048);

      //how to get the private key
      var privKey = csp.ExportParameters(true);

      //and the public key ...
      var pubKey = csp.ExportParameters(false);

      //converting the public key into a string representation
      string pubKeyString;
      {
        //we need some buffer
        var sw = new System.IO.StringWriter();
        //we need a serializer
        var xs = new System.Xml.Serialization.XmlSerializer(typeof(RSAParameters));
        //serialize the key into the stream
        xs.Serialize(sw, pubKey);
        //get the string from the stream
        pubKeyString = sw.ToString();
      }

      //converting it back
      {
        //get a stream from the string
        var sr = new System.IO.StringReader(pubKeyString);
        //we need a deserializer
        var xs = new System.Xml.Serialization.XmlSerializer(typeof(RSAParameters));
        //get the object back from the stream
        pubKey = (RSAParameters)xs.Deserialize(sr);
      }

      //conversion for the private key is no black magic either ... omitted

      //we have a public key ... let's get a new csp and load that key
      csp = new RSACryptoServiceProvider();
      csp.ImportParameters(pubKey);

      //we need some data to encrypt
      var plainTextData = "foobar";

      //for encryption, always handle bytes...
      var bytesPlainTextData = System.Text.Encoding.Unicode.GetBytes(plainTextData);

      //apply pkcs#1.5 padding and encrypt our data 
      var bytesCypherText = csp.Encrypt(bytesPlainTextData, false);

      //we might want a string representation of our cypher text... base64 will do
      var cypherText = Convert.ToBase64String(bytesCypherText);


      /*
       * some transmission / storage / retrieval
       * 
       * and we want to decrypt our cypherText
       */

      //first, get our bytes back from the base64 string ...
      bytesCypherText = Convert.FromBase64String(cypherText);

      //we want to decrypt, therefore we need a csp and load our private key
      csp = new RSACryptoServiceProvider();
      csp.ImportParameters(privKey);

      //decrypt and strip pkcs#1.5 padding
      bytesPlainTextData = csp.Decrypt(bytesCypherText, false);

      //get our original plainText back...
      plainTextData = System.Text.Encoding.Unicode.GetString(bytesPlainTextData);
    }
  }
}

as a side note: the calls to Encrypt() and Decrypt() have a bool parameter that switches between OAEP and PKCS#1.5 padding ... you might want to choose OAEP if it's available in your situation

how get yesterday and tomorrow datetime in c#

You can find this info right in the API reference.

var today = DateTime.Today;
var tomorrow = today.AddDays(1);
var yesterday = today.AddDays(-1);

How can I tell if I'm running in 64-bit JVM or 32-bit JVM (from within a program)?

If you are using JNA, you can check whether com.sun.jna.Native.POINTER_SIZE == 4 (32 bit) or com.sun.jna.Native.POINTER_SIZE == 8 (64 bit).

Django CharField vs TextField

I had an strange problem and understood an unpleasant strange difference: when I get an URL from user as an CharField and then and use it in html a tag by href, it adds that url to my url and that's not what I want. But when I do it by Textfield it passes just the URL that user entered. look at these: my website address: http://myweb.com

CharField entery: http://some-address.com

when clicking on it: http://myweb.comhttp://some-address.com

TextField entery: http://some-address.com

when clicking on it: http://some-address.com

I must mention that the URL is saved exactly the same in DB by two ways but I don't know why result is different when clicking on them

How do I check if the user is pressing a key?

In java you don't check if a key is pressed, instead you listen to KeyEvents. The right way to achieve your goal is to register a KeyEventDispatcher, and implement it to maintain the state of the desired key:

import java.awt.KeyEventDispatcher;
import java.awt.KeyboardFocusManager;
import java.awt.event.KeyEvent;

public class IsKeyPressed {
    private static volatile boolean wPressed = false;
    public static boolean isWPressed() {
        synchronized (IsKeyPressed.class) {
            return wPressed;
        }
    }

    public static void main(String[] args) {
        KeyboardFocusManager.getCurrentKeyboardFocusManager().addKeyEventDispatcher(new KeyEventDispatcher() {

            @Override
            public boolean dispatchKeyEvent(KeyEvent ke) {
                synchronized (IsKeyPressed.class) {
                    switch (ke.getID()) {
                    case KeyEvent.KEY_PRESSED:
                        if (ke.getKeyCode() == KeyEvent.VK_W) {
                            wPressed = true;
                        }
                        break;

                    case KeyEvent.KEY_RELEASED:
                        if (ke.getKeyCode() == KeyEvent.VK_W) {
                            wPressed = false;
                        }
                        break;
                    }
                    return false;
                }
            }
        });
    }
}

Then you can always use:

if (IsKeyPressed.isWPressed()) {
    // do your thing.
}

You can, of course, use same method to implement isPressing("<some key>") with a map of keys and their state wrapped inside IsKeyPressed.

How to redirect back to form with input - Laravel 5

You can try this:

return redirect()->back()->withInput(Input::all())->with('message', 'Something 
went wrong!');

Comparing boxed Long values 127 and 128

Java caches the primitive values from -128 to 127. When we compare two Long objects java internally type cast it to primitive value and compare it. But above 127 the Long object will not get type caste. Java caches the output by .valueOf() method.

This caching works for Byte, Short, Long from -128 to 127. For Integer caching works From -128 to java.lang.Integer.IntegerCache.high or 127, whichever is bigger.(We can set top level value upto which Integer values should get cached by using java.lang.Integer.IntegerCache.high).

 For example:
    If we set java.lang.Integer.IntegerCache.high=500;
    then values from -128 to 500 will get cached and 

    Integer a=498;
    Integer b=499;
    System.out.println(a==b)

    Output will be "true".

Float and Double objects never gets cached.

Character will get cache from 0 to 127

You are comparing two objects. so == operator will check equality of object references. There are following ways to do it.

1) type cast both objects into primitive values and compare

    (long)val3 == (long)val4

2) read value of object and compare

    val3.longValue() == val4.longValue()

3) Use equals() method on object comparison.

    val3.equals(val4);  

What is The difference between ListBox and ListView

A ListView is basically like a ListBox (and inherits from it), but it also has a View property. This property allows you to specify a predefined way of displaying the items. The only predefined view in the BCL (Base Class Library) is GridView, but you can easily create your own.

Another difference is the default selection mode: it's Single for a ListBox, but Extended for a ListView

How can I use JSON data to populate the options of a select box?

Take a look at JQuery view engine and just load the array into a dropdown:

$.ajax({
    url:'suggest.html',
    type:'POST',
    data: 'q=' + str,
    dataType: 'json',
    success: function( json ) {
          // Assumption is that API returned something like:["North","West","South","East"];
          $('#myselect').view(json);
    }
});

See details here: https://jocapc.github.io/jquery-view-engine/docs/ajax-dropdown

How to do if-else in Thymeleaf?

I tried this code to find out if a customer is logged in or anonymous. I did using the th:if and th:unless conditional expressions. Pretty simple way to do it.

<!-- IF CUSTOMER IS ANONYMOUS -->
<div th:if="${customer.anonymous}">
   <div>Welcome, Guest</div>
</div>
<!-- ELSE -->
<div th:unless="${customer.anonymous}">
   <div th:text=" 'Hi,' + ${customer.name}">Hi, User</div>
</div>

Sublime Text 2: How to delete blank/empty lines

To find extra spaces and blank lines press Ctrl+Shift+F Select Regular Expressions Find

[\n\r]{2,}

and then replace with

\n

to remove all kind of spaces in sublime and dreamviewr

The CSRF token is invalid. Please try to resubmit the form

I hade the same issue recently, and my case was something that's not mentioned here yet:

The problem was I was testing it on localhost domain. I'm not sure why exactly was this an issue, but it started to work after I added a host name alias for localhost into /etc/hosts like this:

127.0.0.1        foobar

There's probably something wrong with the session while using Apache and localhost as a domain. If anyone can elaborate in the comments I'd be happy to edit this answer to include more details.

Is there a way to detect if an image is blurry?

Building off of Nike's answer. Its straightforward to implement the laplacian based method with opencv:

short GetSharpness(char* data, unsigned int width, unsigned int height)
{
    // assumes that your image is already in planner yuv or 8 bit greyscale
    IplImage* in = cvCreateImage(cvSize(width,height),IPL_DEPTH_8U,1);
    IplImage* out = cvCreateImage(cvSize(width,height),IPL_DEPTH_16S,1);
    memcpy(in->imageData,data,width*height);

    // aperture size of 1 corresponds to the correct matrix
    cvLaplace(in, out, 1);

    short maxLap = -32767;
    short* imgData = (short*)out->imageData;
    for(int i =0;i<(out->imageSize/2);i++)
    {
        if(imgData[i] > maxLap) maxLap = imgData[i];
    }

    cvReleaseImage(&in);
    cvReleaseImage(&out);
    return maxLap;
}

Will return a short indicating the maximum sharpness detected, which based on my tests on real world samples, is a pretty good indicator of if a camera is in focus or not. Not surprisingly, normal values are scene dependent but much less so than the FFT method which has to high of a false positive rate to be useful in my application.

Running SSH Agent when starting Git Bash on Windows

Put this in your ~/.bashrc (or a file that's source'd from it) which will stop it from being run multiple times unnecessarily per shell:

if [ -z "$SSH_AGENT_PID" ]; then
        eval `ssh-agent -s`
fi

And then add "AddKeysToAgent yes" to ~/.ssh/config:

Host *
    AddKeysToAgent yes

ssh to your server (or git pull) normally and you'll only be asked for password/passphrase once per session.

Should you use .htm or .html file extension? What is the difference, and which file is correct?

It's the same in terms of functionality and support. (most OS recognize both, most Search Engines recognize both)

For my everyday use, I choose .htm because it's shorter to type by 25%.

Custom CSS Scrollbar for Firefox

As of late 2018, there is now limited customization available in Firefox!

See these answers:

And this for background info: https://bugzilla.mozilla.org/show_bug.cgi?id=1460109


There's no Firefox equivalent to ::-webkit-scrollbar and friends.

You'll have to stick with JavaScript.

Plenty of people would like this feature, see: https://bugzilla.mozilla.org/show_bug.cgi?id=77790


As far as JavaScript replacements go, you can try:

What jar should I include to use javax.persistence package in a hibernate based application?

In the latest and greatest Hibernate, I was able to resolve the dependency by including the hibernate-jpa-2.0-api-1.0.0.Final.jar within lib/jpa directory. I didn't find the ejb-persistence jar in the most recent download.

How to do case insensitive string comparison?

As said in recent comments, string::localeCompare supports case insensitive comparisons (among other powerful things).

Here's a simple example

'xyz'.localeCompare('XyZ', undefined, { sensitivity: 'base' }); // returns 0

And a generic function you could use

function equalsIgnoringCase(text, other) {
    return text.localeCompare(other, undefined, { sensitivity: 'base' }) === 0;
}

Note that instead of undefined you should probably enter the specific locale you are working with. This is important as denoted in the MDN docs

in Swedish, ä and a are separate base letters

Sensitivity options

Sensitivity options tabulated from MDN

Browser support

As of time of posting, UC Browser for Android and Opera Mini do not support locale and options parameters. Please check https://caniuse.com/#search=localeCompare for up to date info.

Cloudfront custom-origin distribution returns 502 "ERROR The request could not be satisfied." for some URLs

Beware the Origin Protocol Policy:

For HTTPS viewer requests that CloudFront forwards to this origin, one of the domain names in the SSL certificate on your origin server must match the domain name that you specify for Origin Domain Name. Otherwise, CloudFront responds to the viewer requests with an HTTP status code 502 (Bad Gateway) instead of returning the requested object.

In most cases, you probably want CloudFront to use "HTTP Only", since it fetches objects from a server probably hosted with Amazon too. No need for additional HTTPS complexity at this step.

Note that this is different to the Viewer Protocol Policy. You can read more about the differences between the two here.

Error in spring application context schema

If you don't have control those files, as those files can be part of other projects and you are not authorized to make any changes, then you can bypass those errors in eclipse by going, Preferences --> XML --> XML Files --> Validation --> Referenced file contains errors --> choose Ignore option.

And let project get's validated, the error message will disappear.

What is &#39; and why does Google search replace it with apostrophe?

It's HTML character references for encoding a character by its decimal code point

Look at the ASCII table here and you'll see that 39 (hex 0x27, octal 47) is the code for apostrophe

ASCII table

What is the difference between encode/decode?

mybytestring.encode(somecodec) is meaningful for these values of somecodec:

  • base64
  • bz2
  • zlib
  • hex
  • quopri
  • rot13
  • string_escape
  • uu

I am not sure what decoding an already decoded unicode text is good for. Trying that with any encoding seems to always try to encode with the system's default encoding first.

How do I delete multiple rows with different IDs?

Disclaim: the following suggestion could be an overhead depending on the situation. The function is only tested with MSSQL 2008 R2 but seams be compatible to other versions

if you wane do this with many Id's you may could use a function which creates a temp table where you will be able to DELETE FROM the selection

how the query could look like:

-- not tested
-- @ids will contain a varchar with your ids e.g.'9 12 27 37'
DELETE FROM table WHERE id IN (SELECT i.number FROM iter_intlist_to_tbl(@ids))

here is the function:

ALTER FUNCTION iter_intlist_to_tbl (@list nvarchar(MAX))
   RETURNS @tbl TABLE (listpos int IDENTITY(1, 1) NOT NULL,
                       number  int NOT NULL) AS

   -- funktion gefunden auf http://www.sommarskog.se/arrays-in-sql-2005.html
   -- dient zum übergeben einer liste von elementen

BEGIN
    -- Deklaration der Variablen
    DECLARE @startpos int,
            @endpos   int,
            @textpos  int,
            @chunklen smallint,
            @str      nvarchar(4000),
            @tmpstr   nvarchar(4000),
            @leftover nvarchar(4000)

    -- Startwerte festlegen
   SET @textpos = 1
   SET @leftover = ''

   -- Loop 1
    WHILE @textpos <= datalength(@list) / 2
    BEGIN

        --
        SET @chunklen = 4000 - datalength(@leftover) / 2 --datalength() gibt die anzahl der bytes zurück (mit Leerzeichen)

        --
        SET @tmpstr = ltrim(@leftover + substring(@list, @textpos, @chunklen))--SUBSTRING ( @string ,start , length ) | ltrim(@string) abschneiden aller Leerzeichen am Begin des Strings

        --hochzählen der TestPosition
        SET @textpos = @textpos + @chunklen

        --start position 0 setzen
        SET @startpos = 0

        -- end position bekommt den charindex wo ein [LEERZEICHEN] gefunden wird
        SET @endpos = charindex(' ' COLLATE Slovenian_BIN2, @tmpstr)--charindex(searchChar,Wo,Startposition)

        -- Loop 2
        WHILE @endpos > 0
        BEGIN
            --str ist der string welcher zwischen den [LEERZEICHEN] steht
            SET @str = substring(@tmpstr, @startpos + 1, @endpos - @startpos - 1) 

            --wenn @str nicht leer ist wird er zu int Convertiert und @tbl unter der Spalte 'number' hinzugefügt
            IF @str <> ''
                INSERT @tbl (number) VALUES(convert(int, @str))-- convert(Ziel-Type,Value)

            -- start wird auf das letzte bekannte end gesetzt
            SET @startpos = @endpos

            -- end position bekommt den charindex wo ein [LEERZEICHEN] gefunden wird
            SET @endpos = charindex(' ' COLLATE Slovenian_BIN2, @tmpstr, @startpos + 1)
        END
        -- Loop 2

        -- dient dafür den letzten teil des strings zu selektieren
        SET @leftover = right(@tmpstr, datalength(@tmpstr) / 2 - @startpos)--right(@string,anzahl der Zeichen) bsp.: right("abcdef",3) => "def"
    END
    -- Loop 1

    --wenn @leftover nach dem entfernen aller [LEERZEICHEN] nicht leer ist wird er zu int Convertiert und @tbl unter der Spalte 'number' hinzugefügt
    IF ltrim(rtrim(@leftover)) <> ''
        INSERT @tbl (number) VALUES(convert(int, @leftover))

    RETURN
END


    -- ############################ WICHTIG ############################
    -- das is ein Beispiel wie man die Funktion benutzt
    --
    --CREATE    PROCEDURE get_product_names_iter 
    --      @ids varchar(50) AS
    --SELECT    P.ProductName, P.ProductID
    --FROM      Northwind.Products P
    --JOIN      iter_intlist_to_tbl(@ids) i ON P.ProductID = i.number
    --go
    --EXEC get_product_names_iter '9 12 27 37'
    --
    -- Funktion gefunden auf http://www.sommarskog.se/arrays-in-sql-2005.html
    -- dient zum übergeben einer Liste von Id's
    -- ############################ WICHTIG ############################

Python convert tuple to string

Use str.join:

>>> tup = ('a', 'b', 'c', 'd', 'g', 'x', 'r', 'e')
>>> ''.join(tup)
'abcdgxre'
>>>
>>> help(str.join)
Help on method_descriptor:

join(...)
    S.join(iterable) -> str

    Return a string which is the concatenation of the strings in the
    iterable.  The separator between elements is S.

>>>

What is the difference between Serializable and Externalizable in Java?

The Externalizable interface was not actually provided to optimize the serialization process performance! but to provide means of implementing your own custom processing and offer complete control over the format and contents of the stream for an object and its super types!

Examples of this is the implementation of AMF (ActionScript Message Format) remoting to transfer native action script objects over the network.

Python-Requests close http connection

please use response.close() to close to avoid "too many open files" error

for example:

r = requests.post("https://stream.twitter.com/1/statuses/filter.json", data={'track':toTrack}, auth=('username', 'passwd'))
....
r.close()

MVC Razor Hidden input and passing values

If you are using Razor, you cannot access the field directly, but you can manage its value.

The idea is that the first Microsoft approach drive the developers away from Web Development and make it easy for Desktop programmers (for example) to make web applications.

Meanwhile, the web developers, did not understand this tricky strange way of ASP.NET.

Actually this hidden input is rendered on client-side, and the ASP has no access to it (it never had). However, in time you will see its a piratical way and you may rely on it, when you get use with it. The web development differs from the Desktop or Mobile.

The model is your logical unit, and the hidden field (and the whole view page) is just a representative view of the data. So you can dedicate your work on the application or domain logic and the view simply just serves it to the consumer - which means you need no detailed access and "brainstorming" functionality in the view.

The controller actually does work you need for manage the hidden or general setup. The model serves specific logical unit properties and functionality and the view just renders it to the end user, simply said. Read more about MVC.

Model

public class MyClassModel
{
    public int Id { get; set; }
    public string Name { get; set; }
    public string MyPropertyForHidden { get; set; }
}

This is the controller aciton

public ActionResult MyPageView()
{
    MyClassModel model = new MyClassModel(); // Single entity, strongly-typed
    // IList model = new List<MyClassModel>(); // or List, strongly-typed
    // ViewBag.MyHiddenInputValue = "Something to pass"; // ...or using ViewBag

    return View(model);
}

The view is below

//This will make a Model property of the View to be of MyClassModel
@model MyNamespace.Models.MyClassModel // strongly-typed view
// @model IList<MyNamespace.Models.MyClassModel> // list, strongly-typed view

// ... Some Other Code ...

@using(Html.BeginForm()) // Creates <form>
{
    // Renders hidden field for your model property (strongly-typed)
    // The field rendered to server your model property (Address, Phone, etc.)
    Html.HiddenFor(model => Model.MyPropertyForHidden); 

    // For list you may use foreach on Model
    // foreach(var item in Model) or foreach(MyClassModel item in Model)
}

// ... Some Other Code ...

The view with ViewBag:

// ... Some Other Code ...

@using(Html.BeginForm()) // Creates <form>
{
    Html.Hidden(
        "HiddenName",
        ViewBag.MyHiddenInputValue,
        new { @class = "hiddencss", maxlength = 255 /*, etc... */ }
    );
}

// ... Some Other Code ...

We are using Html Helper to render the Hidden field or we could write it by hand - <input name=".." id=".." value="ViewBag.MyHiddenInputValue"> also.

The ViewBag is some sort of data carrier to the view. It does not restrict you with model - you can place whatever you like.

Escaping single quotes in JavaScript string for JavaScript evaluation

Best to use JSON.stringify() to cover all your bases, like backslashes and other special characters. Here's your original function with that in place instead of modifying strInputString:

function testEscape() {
    var strResult = "";
    var strInputString = "fsdsd'4565sd";

    var strTest = "strResult = " + JSON.stringify(strInputString) + ";";
    eval(strTest);
    alert(strResult);
}

(This way your strInputString could be something like \\\'\"'"''\\abc'\ and it will still work fine.)

Note that it adds its own surrounding double-quotes, so you don't need to include single quotes anymore.

Reset Windows Activation/Remove license key

On Windows XP -

  1. Reboot into "Safe mode with Command Prompt"
  2. Type "explorer" in the command prompt that comes up and push [Enter]
  3. Click on Start>Run, and type the following :

    rundll32.exe syssetup,SetupOobeBnk

Reboot, and login as normal.

This will reset the 30 day timer for activation back to 30 days so you can enter in the key normally.

how to set "camera position" for 3d plots using python/matplotlib?

Try the following code to find the optimal camera position

Move the viewing angle of the plot using the keyboard keys as mentioned in the if clause

Use print to get the camera positions

def move_view(event):
    ax.autoscale(enable=False, axis='both') 
    koef = 8
    zkoef = (ax.get_zbound()[0] - ax.get_zbound()[1]) / koef
    xkoef = (ax.get_xbound()[0] - ax.get_xbound()[1]) / koef
    ykoef = (ax.get_ybound()[0] - ax.get_ybound()[1]) / koef
    ## Map an motion to keyboard shortcuts
    if event.key == "ctrl+down":
        ax.set_ybound(ax.get_ybound()[0] + xkoef, ax.get_ybound()[1] + xkoef)
    if event.key == "ctrl+up":
        ax.set_ybound(ax.get_ybound()[0] - xkoef, ax.get_ybound()[1] - xkoef)
    if event.key == "ctrl+right":
        ax.set_xbound(ax.get_xbound()[0] + ykoef, ax.get_xbound()[1] + ykoef)
    if event.key == "ctrl+left":
        ax.set_xbound(ax.get_xbound()[0] - ykoef, ax.get_xbound()[1] - ykoef)
    if event.key == "down":
        ax.set_zbound(ax.get_zbound()[0] - zkoef, ax.get_zbound()[1] - zkoef)
    if event.key == "up":
        ax.set_zbound(ax.get_zbound()[0] + zkoef, ax.get_zbound()[1] + zkoef)
    # zoom option
    if event.key == "alt+up":
        ax.set_xbound(ax.get_xbound()[0]*0.90, ax.get_xbound()[1]*0.90)
        ax.set_ybound(ax.get_ybound()[0]*0.90, ax.get_ybound()[1]*0.90)
        ax.set_zbound(ax.get_zbound()[0]*0.90, ax.get_zbound()[1]*0.90)
    if event.key == "alt+down":
        ax.set_xbound(ax.get_xbound()[0]*1.10, ax.get_xbound()[1]*1.10)
        ax.set_ybound(ax.get_ybound()[0]*1.10, ax.get_ybound()[1]*1.10)
        ax.set_zbound(ax.get_zbound()[0]*1.10, ax.get_zbound()[1]*1.10)
    
    # Rotational movement
    elev=ax.elev
    azim=ax.azim
    if event.key == "shift+up":
        elev+=10
    if event.key == "shift+down":
        elev-=10
    if event.key == "shift+right":
        azim+=10
    if event.key == "shift+left":
        azim-=10

    ax.view_init(elev= elev, azim = azim)

    # print which ever variable you want 

    ax.figure.canvas.draw()

fig.canvas.mpl_connect("key_press_event", move_view)

plt.show()

Printing with "\t" (tabs) does not result in aligned columns

As mentioned by other folks, the variable length of the string is the issue.

Rather than reinventing the wheel, Apache Commons has a nice, clean solution for this in StringUtils.

StringUtils.rightPad("String to extend",100); //100 is the length you want to pad out to.

Get the date of next monday, tuesday, etc

See strtotime()

strtotime('next tuesday');

You could probably find out if you have gone past that day by looking at the week number:

$nextTuesday = strtotime('next tuesday');
$weekNo = date('W');
$weekNoNextTuesday = date('W', $nextTuesday);

if ($weekNoNextTuesday != $weekNo) {
    //past tuesday
}

Function pointer to member function

While you unfortunately cannot convert an existing member function pointer to a plain function pointer, you can create an adapter function template in a fairly straightforward way that wraps a member function pointer known at compile-time in a normal function like this:

template <class Type>
struct member_function;

template <class Type, class Ret, class... Args>
struct member_function<Ret(Type::*)(Args...)>
{
    template <Ret(Type::*Func)(Args...)>
    static Ret adapter(Type &obj, Args&&... args)
    {
        return (obj.*Func)(std::forward<Args>(args)...);
    }
};

template <class Type, class Ret, class... Args>
struct member_function<Ret(Type::*)(Args...) const>
{
    template <Ret(Type::*Func)(Args...) const>
    static Ret adapter(const Type &obj, Args&&... args)
    {
        return (obj.*Func)(std::forward<Args>(args)...);
    }
};

 

int (*func)(A&) = &member_function<decltype(&A::f)>::adapter<&A::f>;

Note that in order to call the member function, an instance of A must be provided.

How do I insert values into a Map<K, V>?

The syntax is

data.put("John","Taxi driver");

Adding a month to a date in T SQL

Use DATEADD:

DATEADD(month, 1, reference_dt)

Pandas Replace NaN with blank/empty string

Use a formatter, if you only want to format it so that it renders nicely when printed. Just use the df.to_string(... formatters to define custom string-formatting, without needlessly modifying your DataFrame or wasting memory:

df = pd.DataFrame({
    'A': ['a', 'b', 'c'],
    'B': [np.nan, 1, np.nan],
    'C': ['read', 'unread', 'read']})
print df.to_string(
    formatters={'B': lambda x: '' if pd.isnull(x) else '{:.0f}'.format(x)})

To get:

   A B       C
0  a      read
1  b 1  unread
2  c      read

Installing Oracle Instant Client

The directions state:

  1. Download the appropriate Instant Client packages for your platform. All installations REQUIRE the Basic package.
  2. Unzip the packages into a single directory such as "instantclient".
  3. Set the library loading path in your environment to the directory in Step 2 ("instantclient"). On many UNIX platforms, LD_LIBRARY_PATH is the appropriate environment variable. On Windows, PATH should be used.
  4. Start your application and enjoy.

Suggest extracting/unzipping into a new directory. They've suggested instantclient, but you can name the directory anything you like. Name it C:\OracleInstantClient\ if you choose.

Then in Step 3, open a Windows Command Prompt. Type:

PATH C:\OracleInstantClient; %PATH%`

That's all there is to it!

How to get object size in memory?

Unmanaged object:

  • Marshal.SizeOf(object yourObj);

Value Types:

  • sizeof(object val)

Managed object:

Can I change the checkbox size using CSS?

My reputation is slightly too low to post comments, but I made a modification to Jack Miller's code above in order to get it to not change size when you check and uncheck it. This was causing text alignment problems for me.

_x000D_
_x000D_
    input[type=checkbox] {
        width: 17px;
        -webkit-appearance: none;
        -moz-appearance: none;
        height: 17px;
        border: 1px solid black;
    }

    input[type=checkbox]:checked {
        background-color: #F58027;
    }

    input[type=checkbox]:checked:after {
        margin-left: 4px;
        margin-top: -1px;
        width: 4px;
        height: 12px;
        border: solid white;
        border-width: 0 2px 2px 0;
        -webkit-transform: rotate(45deg);
        -moz-transform: rotate(45deg);
        -ms-transform: rotate(45deg);
        transform: rotate(45deg);
        content: "";
        display: inline-block;
    }
    input[type=checkbox]:after {
        margin-left: 4px;
        margin-top: -1px;
        width: 4px;
        height: 12px;
        border: solid white;
        border-width: 0;
        -webkit-transform: rotate(45deg);
        -moz-transform: rotate(45deg);
        -ms-transform: rotate(45deg);
        transform: rotate(45deg);
        content: "";
        display: inline-block;
    }
_x000D_
<label><input type="checkbox"> Test</label>
_x000D_
_x000D_
_x000D_

Use component from another module

Whatever you want to use from another module, just put it in the export array. Like this-

 @NgModule({
  declarations: [TaskCardComponent],
  exports: [TaskCardComponent],
  imports: [MdCardModule]
})

Colorized grep -- viewing the entire file with highlighted matches

I use following command for similar purpose:

grep -C 100 searchtext file

This will say grep to print 100 * 2 lines of context, before & after of the highlighted search text.

Should I always use a parallel stream when possible?

JB hit the nail on the head. The only thing I can add is that Java 8 doesn't do pure parallel processing, it does paraquential. Yes I wrote the article and I've been doing F/J for thirty years so I do understand the issue.

How to select multiple rows filled with constants?

The following bare VALUES command works for me in PostgreSQL:

VALUES (1,2,3), (4,5,6), (7,8,9)

How to open a new HTML page using jQuery?

You need to use ajax.

http://api.jquery.com/jQuery.ajax/

<code>
$.ajax({
  url: 'ajax/test.html',
  success: function(data) {
    $('.result').html(data);
    alert('Load was performed.');
  }
});
</code>